Nothing Special   »   [go: up one dir, main page]

Study Guide gr04 2020

Download as pdf or txt
Download as pdf or txt
You are on page 1of 152

Study/Resource Guide

Georgia
Milestones Assessment System

Study/Resource Guide
for Students and Parents
Grade 4

The Study/Resource Guides are intended to serve as a resource for parents and students. They
contain practice questions and learning activities for each content area. The standards identified
in the Study/Resource Guides address a sampling of the state-mandated content standards.
For the purposes of day-to-day classroom instruction, teachers should consult the wide array of
resources that can be found at www.georgiastandards.org.

Copyright © 2020 by Georgia Department of Education. All rights reserved.


Table of Contents

THE GEORGIA MILESTONES ASSESSMENT SYSTEM  . . . . . . . . . . . . . . . . . . . . . . . . . . . . . . . . . . .  3


HOW TO USE THIS GUIDE  . . . . . . . . . . . . . . . . . . . . . . . . . . . . . . . . . . . . . . . . . . . . . . . . . . . . . . . .  4
PREPARING FOR TAKING TESTS  . . . . . . . . . . . . . . . . . . . . . . . . . . . . . . . . . . . . . . . . . . . . . . . . . . .  5
OVERVIEW OF THE END-OF-GRADE ASSESSMENT  . . . . . . . . . . . . . . . . . . . . . . . . . . . . . . . . . . . . .  6
TYPES OF ITEMS  . . . . . . . . . . . . . . . . . . . . . . . . . . . . . . . . . . . . . . . . . . . . . . . . . . . . . . . . . .  6
DEPTH OF KNOWLEDGE  . . . . . . . . . . . . . . . . . . . . . . . . . . . . . . . . . . . . . . . . . . . . . . . . . . . . . . . . .  9
ENGLISH LANGUAGE ARTS (ELA)  . . . . . . . . . . . . . . . . . . . . . . . . . . . . . . . . . . . . . . . . . . . . . . . . .  12
DESCRIPTION OF TEST FORMAT AND ORGANIZATION  . . . . . . . . . . . . . . . . . . . . . . . . . . . . . .  12
CONTENT   . . . . . . . . . . . . . . . . . . . . . . . . . . . . . . . . . . . . . . . . . . . . . . . . . . . . . . . . . . . . . .  12
ITEM TYPES  . . . . . . . . . . . . . . . . . . . . . . . . . . . . . . . . . . . . . . . . . . . . . . . . . . . . . . . . . . . . .  12
ENGLISH LANGUAGE ARTS (ELA) DEPTH OF KNOWLEDGE EXAMPLE ITEMS  . . . . . . . . . . . . .  13
ENGLISH LANGUAGE ARTS (ELA) CONTENT DESCRIPTION AND ADDITIONAL SAMPLE ITEMS  .  25
ENGLISH LANGUAGE ARTS (ELA) ADDITIONAL SAMPLE ITEM KEYS  . . . . . . . . . . . . . . . . . . . .  68
ENGLISH LANGUAGE ARTS (ELA) EXAMPLE SCORING RUBRICS AND EXEMPLAR RESPONSES  75
ENGLISH LANGUAGE ARTS (ELA) WRITING RUBRICS  . . . . . . . . . . . . . . . . . . . . . . . . . . . . . . .  84
MATHEMATICS  . . . . . . . . . . . . . . . . . . . . . . . . . . . . . . . . . . . . . . . . . . . . . . . . . . . . . . . . . . . . . . .  94
DESCRIPTION OF TEST FORMAT AND ORGANIZATION  . . . . . . . . . . . . . . . . . . . . . . . . . . . . . .  94
CONTENT  . . . . . . . . . . . . . . . . . . . . . . . . . . . . . . . . . . . . . . . . . . . . . . . . . . . . . . . . . . . . . . .  94
ITEM TYPES  . . . . . . . . . . . . . . . . . . . . . . . . . . . . . . . . . . . . . . . . . . . . . . . . . . . . . . . . . . . . .  94
MATHEMATICS DEPTH OF KNOWLEDGE EXAMPLE ITEMS  . . . . . . . . . . . . . . . . . . . . . . . . . . .  95
MATHEMATICS CONTENT DESCRIPTION AND ADDITIONAL SAMPLE ITEMS  . . . . . . . . . . . . .  101
MATHEMATICS ADDITIONAL SAMPLE ITEM KEYS  . . . . . . . . . . . . . . . . . . . . . . . . . . . . . . . .  134
MATHEMATICS EXAMPLE SCORING RUBRICS AND EXEMPLAR RESPONSES  . . . . . . . . . . . .  139
APPENDIX: LANGUAGE PROGRESSIVE SKILLS, BY GRADE  . . . . . . . . . . . . . . . . . . . . . . . . . . . .  151

Copyright © 2020 by Georgia Department of Education. All rights reserved.


The Georgia Milestones Assessment System

THE GEORGIA MILESTONES ASSESSMENT SYSTEM

Dear Student,
This Georgia Milestones Grade 4 Study/Resource Guide for Students
and Parents is intended as a resource for parents and students. It
contains sample questions and helpful activities to give you an idea
of what test questions look like on Georgia Milestones and what the
Grade 4 End-of-Grade (EOG) assessment covers.
These sample questions are fully explained and will tell you why each
answer is either correct or incorrect.

Get ready—open this guide—and get started!

Georgia Milestones Grade 4 EOG Study/Resource Guide for Students and Parents Page 3 of 152
Copyright © 2020 by Georgia Department of Education. All rights reserved.
How to Use This Guide

HOW TO USE THIS GUIDE

Let’s get started!


✽ Get it together!
• This guide
• Pen or pencil
• Highlighter
• Paper

✽ Gather materials
• Classroom notebooks
• Textbooks

✽ Study space
• Find a comfortable place to sit .
• Use good lighting .
• Time to focus—no TV, games, or phones!

✽ Study time
• Set aside some time after school .
• Set a goal—how long are you going to study?
• Remember—you cannot do this all at one time .
• Study a little at a time every day .

✽ Study buddy
• Work with a friend, sister, brother, parent—anyone who can help!
• Ask questions—it is better to ask now and get answers .
• Make sure you know what you need to do—read the directions before you start .
• Ask your teacher if you need help .

✽ Test-taking help
• Read each question and all of the answer choices carefully .
• Be neat—use scratch paper .
• Check your work!

Page 4 of 152 Georgia Milestones Grade 4 EOG Study/Resource Guide for Students and Parents
Copyright © 2020 by Georgia Department of Education. All rights reserved.
Preparing for Taking Tests

PREPARING FOR TAKING TESTS

Getting ready!

Here are some ideas to think about before you take a test .
• Get plenty of rest and eat right . Take care of your body and your mind will do the rest .

• If you are worried about a test, don’t be . Talk with a teacher, parent, or friend about what is
expected of you .

• Review the things you have learned all year long . Feel good about it .

• Remember that a test is just one look at what you know . Your class work, projects, and other tests
will also show your teachers how much you have learned throughout the year .

Try your best!

Georgia Milestones Grade 4 EOG Study/Resource Guide for Students and Parents Page 5 of 152
Copyright © 2020 by Georgia Department of Education. All rights reserved.
Overview of the End-of-Grade Assessment

OVERVIEW OF THE END-OF-GRADE ASSESSMENT

What is on the End-of-Grade Assessment?


✽✽ English Language Arts (ELA)
✽✽ Mathematics

TYPES OF ITEMS
✽✽ Selected-response items—also called multiple-choice questions
•• Appear in English Language Arts (ELA) and Mathematics
•• There is a question, problem, or statement that is followed by four answer choices.
•• There is only ONE right answer, so read EACH answer choice carefully.
•• Eliminate the answers that you know are wrong.
•• Look for the answer that is the BEST choice.

✽✽ Technology-enhanced items
•• These types of items appear in English Language Arts (ELA) and Mathematics and are worth one or
two points. Partial credit may be awarded on two-point items if you select some but not all of the
correct answers or if you get one part of the question correct but not the other part.
•• There is a question, problem, or statement.
•• Read the directions for each item carefully.
•• Eliminate the answers you know are wrong.
•• In multi-select items, you will be asked to select more than one right answer.
•• In multi-part items, the items will have more than one part. You will need to provide an answer in
each part. Sometimes, the different parts of the items are dependent on each other.
•• In evidence-based selected-response (EBSR) items, you will be asked to answer the first part of the
question. Then, you will answer the second part of the question based on the answer to the first
part.
•• In drag-and-drop items, you will be asked to use a mouse, touchpad, or touchscreen to move
responses to designated areas on the screen.
•• In drop-down menu items, you will be asked to use a mouse, touchpad, or touchscreen to open a
drop-down menu and select an option from the menu. A drop-down menu item may have more than
one drop-down menu.
•• In keypad-input items, you will be asked to use a physical keyboard or the pop-up keyboard on a
touchscreen to type a number, expression, or equation into an answer box.
•• In coordinate-graph items, you will be asked to use a mouse, touchpad, or touchscreen to draw
lines and/or plot points on a coordinate grid on the screen.
•• In line-plot items, you will be asked to use a mouse, touchpad, or touchscreen to place Xs above a
number line to create a line plot.

Page 6 of 152 Georgia Milestones Grade 4 EOG Study/Resource Guide for Students and Parents
Copyright © 2020 by Georgia Department of Education. All rights reserved.
Overview of the End-of-Grade Assessment

•• In bar-graph items, you will be asked to use a mouse, touchpad, or touchscreen to select the height
of each bar to create a bar graph.
•• In number-line items, you will be asked to use a mouse, touchpad, or touchscreen to plot a point
and/or represent inequalities.
•• Since some technology-enhanced items in this guide were designed to be used in an online,
interactive-delivery format, some of the item-level directions will not appear to be applicable when
working within the format presented in this document (for example, “Move the characteristics into
boxes,” “Create a scatter plot,” or “Click To Respond”).
•• This icon  identifies special directions that will help you answer technology-enhanced items
as shown in the format presented within this guide. These directions do not appear in the
online version of the test but explain information about how the item works that would be easily
identifiable if you were completing the item in an online environment.
•• To practice using technology-enhanced items in an online environment very similar to how they will
appear on the online test, visit “Experience Online Testing Georgia.”
1. Go to the website “Welcome to Experience Online Testing Georgia”
(http://gaexperienceonline.com/).
2. Select “Test Practice.”
3. On the right side of the page, you will see “End-of-Grade (EOG) Spring Main.” Select “Online
Tools Training” which appears underneath it.
4. Select “EOG Test Practice.”
5. Select “Technology Enhanced Items.”
6. Select “All Grades.”
7. You will be taken to a login screen. Use the username and password provided on the screen to
log in and practice navigating technology-enhanced items online.

Please note that Google Chrome is the only supported browser for this public version of the online
testing environment.

✽✽ Constructed-response items
•• Appear in English Language Arts (ELA) only
•• There is a question, problem, or statement but no answer choices.
•• Read the question carefully and think about what you are asked to do.
•• You must write your answer.
•• Go back to the passage to look for details and information.
•• You will be scored on accuracy and how well you support your answer with evidence.

✽✽ Extended constructed-response items


•• Appear in English Language Arts (ELA) only
•• These are similar to the constructed-response items.
•• Sometimes they have more than one part, or they require a longer answer.
•• Check that you have answered all parts of the question.

Georgia Milestones Grade 4 EOG Study/Resource Guide for Students and Parents Page 7 of 152
Copyright © 2020 by Georgia Department of Education. All rights reserved.
Overview of the End-of-Grade Assessment

✽✽ Extended writing-response
•• Appears in English Language Arts (ELA) only
•• There is a question, problem, or statement.
•• You may be asked to do more than one thing.
•• You will be asked to read two passages and then write an essay.
•• You will be scored on how well you answer the question and the quality of your writing.
•• Organize your ideas clearly.
•• Use correct grammar, punctuation, and spelling.
•• Support your answer with evidence from the text.

Page 8 of 152 Georgia Milestones Grade 4 EOG Study/Resource Guide for Students and Parents
Copyright © 2020 by Georgia Department of Education. All rights reserved.
Depth of Knowledge

DEPTH OF KNOWLEDGE

Test questions are designed with a Depth of Knowledge (DOK) level in mind. As you go from Level 1 to
Level 4, the items get more and more challenging. They take more thinking and reasoning to answer. You
may have experienced these types of questions in your classroom as your teachers find ways to challenge
you each day.
A Level 1 item may not require as much thinking as a Level 4 item—but that does not mean it’s easy.
A Level 4 item may have more than one part or ask you to write something.
Here is some information to help you understand just what a DOK level really is.

Level 1 (Recall of Information)


✽✽ Identify, list, or define something.
✽✽ Questions may start with who, what, when, and where.
✽✽ Recall facts, terms, or identify information.

Level 2 (Basic Reasoning)


✽✽ Think about things—it is more than just remembering something.
✽✽ Describe or explain something.
✽✽ Answer the questions “how” or “why.”

Level 3 (Complex Reasoning)


✽✽ Go beyond explaining or describing “how and why.”
✽✽ Explain or justify your answers.
✽✽ Give reasons and evidence for your response.
✽✽ Make connections and explain a concept or a “big idea.”

Level 4 (Extended Reasoning)


✽✽ Complex thinking required!
✽✽ Plan, investigate, or apply a deeper understanding.
✽✽ These items will take more time to write.
✽✽ Connect and relate ideas.
✽✽ Show evidence by doing a task, creating a product, or writing a response.

Georgia Milestones Grade 4 EOG Study/Resource Guide for Students and Parents Page 9 of 152
Copyright © 2020 by Georgia Department of Education. All rights reserved.
Depth of Knowledge

Depth of Knowledge

Level 1—Recall of Information


Level 1 asks you to identify, list, or define. You may be asked to recall who, what, when, and where.
You may also be asked to recall facts and terms or identify information in documents, quotations,
maps, charts, tables, graphs, or illustrations. Items that ask you to “describe” and/or “explain” could
be Level 1 or Level 2. A Level 1 item requires that you just recall, recite, or repeat information.
Skills Demonstrated Question Cues
•• Make observations •• Tell who, what, when, or where
•• Recall information •• Find
•• Recognize formulas, properties, patterns, •• List
processes •• Define
•• Know vocabulary, definitions •• Identify; label; name
•• Know basic concepts •• Choose; select
•• Perform one-step processes •• Compute; estimate
•• Translate from one representation to another •• Express as
•• Identify relationships •• Read from data displays
•• Order

Level 2—Basic Reasoning


Level 2 includes some thinking that goes beyond recalling or repeating a response. A Level 2
“describe” and/or “explain” item would require that you go beyond a description or explanation of
information to describe and/or explain a result or “how” or “why.”
Skills Demonstrated Question Cues
•• Apply learned information to abstract and real-life •• Apply
situations •• Calculate; solve
•• Use methods, concepts, and theories in abstract •• Complete
and real-life situations
•• Describe
•• Perform multi-step processes
•• Explain how; demonstrate
•• Solve problems using required skills or knowledge
•• Construct data displays
(requires more than habitual response)
•• Construct; draw
•• Make a decision about how to proceed
•• Analyze
•• Identify and organize components of a whole
•• Extend
•• Extend patterns
•• Connect
•• Identify/describe cause and effect
•• Classify
•• Make basic inferences or logical predictions from
data or text •• Arrange
•• Interpret facts •• Compare; contrast
•• Compare or contrast simple concepts/ideas •• Predict

Page 10 of 152 Georgia Milestones Grade 4 EOG Study/Resource Guide for Students and Parents
Copyright © 2020 by Georgia Department of Education. All rights reserved.
Depth of Knowledge

Level 3—Complex Reasoning


Level 3 requires reasoning, using evidence, and thinking on a higher level than Level 1 and Level 2.
You will go beyond explaining or describing “how and why” to justifying the “how and why” through
reasons and evidence. Level 3 items often involve making connections across time and place to
explain a concept or a “big idea.”
Skills Demonstrated Question Cues
•• Solve an open-ended problem with more than •• Plan; prepare
one correct answer •• Create; design
•• Create a pattern •• Ask “what if?” questions
•• Generalize from given facts •• Generalize
•• Relate knowledge from several sources •• Justify; explain why; support; convince
•• Draw conclusions •• Assess
•• Translate knowledge into new contexts •• Rank; grade
•• Compare and discriminate between ideas •• Test; judge
•• Assess value of methods, concepts, theories, •• Recommend
processes, and formulas
•• Select
•• Make choices based on a reasoned argument
•• Conclude
•• Verify the value of evidence, information,
numbers, and data

Level 4—Extended Reasoning


Level 4 requires the complex reasoning of Level 3 with the addition of planning, investigating,
applying deeper understanding, and/or developing that will require a longer period of time. You may
be asked to connect and relate ideas and concepts within the content area or among content areas
in order to be at this highest level. The Level 4 items would be a show of evidence—through a task, a
product, or an extended response—that the higher-level demands have been met.
Skills Demonstrated Question Cues
•• Analyze and synthesize information from •• Design
multiple sources •• Connect
•• Examine and explain alternative perspectives •• Synthesize
across a variety of sources
•• Apply concepts
•• Describe and illustrate how common themes
•• Critique
are found across texts from different cultures
•• Analyze
•• Apply mathematical models to illuminate a
problem or situation •• Create
•• Design a mathematical model to inform and •• Prove
solve a practical or abstract situation
•• Combine and synthesize ideas into new
concepts

Georgia Milestones Grade 4 EOG Study/Resource Guide for Students and Parents Page 11 of 152
Copyright © 2020 by Georgia Department of Education. All rights reserved.
English Language Arts (ELA)

ENGLISH LANGUAGE ARTS (ELA)

DESCRIPTION OF TEST FORMAT AND ORGANIZATION


The Grade 4 English Language Arts (ELA) EOG assessment has a total of 51 items.
You will answer a variety of item types on the test. Some of the items are selected-response (multiple-
choice), which means you choose the correct answer from four choices. Some items will ask you to write
your response using details from the text. There will also be a writing prompt that will ask you to write an
essay.
The test will be given in three sections.
•• Section 1 will be given on Day 1. You will be given a maximum of 90 minutes to complete the section.
•• Sections 2 and 3 will be given over one or two days. You may have up to 80 minutes to complete each
section.

CONTENT
The Grade 4 English Language Arts (ELA) EOG assessment will measure the Grade 4 standards that are
described at www.georgiastandards.org.
The content of the assessment covers standards that are reported under these domains:
•• Reading and Vocabulary
•• Writing and Language
There are two kinds of texts—literary and informational text.
There are two kinds of essays students may be asked to write—an opinion essay and an informational or
explanatory essay.
Students will also write an extended constructed-response using narrative techniques. Students may be
asked to continue a story or perhaps write a new beginning, for example. The stimulus text may be literary
or informational. (Item 8 on page 34 gives an example of a prompt that requires a narrative response.)

ITEM TYPES
The English Language Arts (ELA) portion of the Grade 4 EOG assessment consists of selected-response
(multiple-choice), technology-enhanced, constructed-response, extended constructed-response, and
extended writing-response items.

Page 12 of 152 Georgia Milestones Grade 4 EOG Study/Resource Guide for Students and Parents
Copyright © 2020 by Georgia Department of Education. All rights reserved.
English Language Arts (ELA)

ENGLISH LANGUAGE ARTS (ELA) DEPTH OF KNOWLEDGE EXAMPLE ITEMS


Example items that represent applicable DOK levels are provided for you on the following pages. The items
and explanations of what is expected of you to answer them will help you prepare for the test.
All example and sample items contained in this guide are the property of the Georgia Department of
Education.
Example Item 1
Selected-Response
DOK Level 1: This is a DOK level 1 item because it requires the student to distinguish between common
and proper nouns.
English Language Arts (ELA) Grade 4 Content Domain II: Writing and Language
Standard: ELAGSE4L2a. Demonstrate command of the conventions of standard English capitalization,
punctuation, and spelling when writing.
a. Use correct capitalization.

Read the sentence.

My mother picked out our next family car from a dealership in texas.

Which underlined word in the sentence should start with a capital letter?

A. mother
B. family
C. dealership
D. texas

Correct Answer: D
Explanation of Correct Answer: The correct answer is choice (D) texas. Cities, towns, states, and nations
are always capitalized. Choice (A) is incorrect because it is not used as a name. Choices (B) and (C) are
incorrect because they are common nouns.

Georgia Milestones Grade 4 EOG Study/Resource Guide for Students and Parents Page 13 of 152
Copyright © 2020 by Georgia Department of Education. All rights reserved.
English Language Arts (ELA)

Read the article and answer example items 2 and 3.

Central Park
Before 1850, many of the world’s great cities had nice parks. However, there were no city parks in
the United States. New York City was a busy city, and there were no places to escape from the noise
or from the smell of horses. Some important people in New York City decided that a park was needed.
The city had a contest to see who could design the best park.

There were many different designs for the park. People argued about the purpose of the park. Some
people said that it should be like parks in England and France. Those parks were mostly for people
who had lots of money. The parks had long, straight roads. People who could afford horses and
carriages could ride in the parks. The gardens in those parks were very square. They had lots of large
stone buildings. The parks were built like the gardens around palaces.

Other people said that a park should be designed for all the people, not just the rich. That meant the
park should be good for walking, and there shouldn’t be long, straight roads. Straight roads and big
buildings allowed for less natural scenery.

The plan that the city chose was more like a park for all the people. It included large green areas and
curvy walking paths. These paths were built around natural features, like large rocks. The park had
very few buildings. It had special paths for horses to keep the animals separate from people. Today,
Central Park is considered one of the greatest parks in the world.

Page 14 of 152 Georgia Milestones Grade 4 EOG Study/Resource Guide for Students and Parents
Copyright © 2020 by Georgia Department of Education. All rights reserved.
English Language Arts (ELA)

Example Item 2
Selected-Response
DOK Level 2: This is a DOK level 2 item because the student is asked to apply knowledge of the text in
order to answer the question.
English Language Arts (ELA) Grade 4 Content Domain I: Reading and Vocabulary
Genre: Informational
Standard: ELAGSE4RI3. Explain events, procedures, ideas, or concepts in a historical, scientific, or
technical text, including what happened and why, based on specific information in the text.

Which sentence BEST describes why Central Park was designed to have few straight roads?

A. Curved roads were better for horses.


B. More natural features were left in place.
C. The builders used roads that already existed.
D. The roads were built to go around the gardens.

Correct Answer: B
Explanation of Correct Answer: The correct answer is choice (B) More natural features were left in place.
The park was built to show as much natural scenery as possible, so roads curved around existing rocks
and other features. Choice (A) is incorrect because the author does not tell you curved roads are better
for horses. Choice (C) is incorrect because the author never says this. Choice (D) is incorrect because the
author never mentions gardens in Central Park.

Georgia Milestones Grade 4 EOG Study/Resource Guide for Students and Parents Page 15 of 152
Copyright © 2020 by Georgia Department of Education. All rights reserved.
English Language Arts (ELA)

Example Item 3
Drag-and-Drop Technology-Enhanced
DOK Level 3: This is a DOK level 3 item because students are asked to draw a conclusion based on
information in the article and then support that conclusion with evidence from the article.
English Language Arts (ELA) Grade 4 Content Domain I: Reading and Vocabulary
Genre: Informational
Standard: ELAGSE4RI3. Explain events, procedures, ideas, or concepts in a historical, scientific, or
technical text, including what happened and why, based on specific information in the text.

ÂÂ
Due to the size of the response area, this item has a “Click To Respond” button on the screen.
Clicking this button will bring up the response area at full size.

Go on to the next page to finish example item 3.

Page 16 of 152 Georgia Milestones Grade 4 EOG Study/Resource Guide for Students and Parents
Copyright © 2020 by Georgia Department of Education. All rights reserved.
English Language Arts (ELA)

Example Item 3. Continued.

ÂÂ
Use a mouse, touchpad, or touchscreen to move the descriptions below the chart into the chart next
to the bullets.

Georgia Milestones Grade 4 EOG Study/Resource Guide for Students and Parents Page 17 of 152
Copyright © 2020 by Georgia Department of Education. All rights reserved.
English Language Arts (ELA)

Example Item 3. Continued.


Scoring Rubric

Points Description
2 The student correctly fills in both columns (order within each column does not matter).
The student correctly fills in the first column or the first column and one bullet in the
1
second column (order within each column does not matter).
0 The student correctly fills in the second column or does not correctly fill in either column.

Exemplar Response
The correct response is shown below.

The correct response in the first column is “The park was designed with paths built around natural features
so that it could be used by everyone.” This is the correct response because the article states that Central
Park’s design was chosen to appeal to all people. Therefore, the greenery and trails could be utilized and
enjoyed by everyone, not just by wealthy people who could travel by horse and carriage. One of the correct
responses for the second column is “That meant the park should be good for walking, and there shouldn’t
be long, straight roads.” The other correct response for the second column is “It included large green
areas and curvy walking paths.” These are the correct responses for the second column because they are
details that reinforce the idea that a park designed for all people would be one that people could enjoy
simply by walking around the park. NOTE: The order of the details in the second column does not matter.

Page 18 of 152 Georgia Milestones Grade 4 EOG Study/Resource Guide for Students and Parents
Copyright © 2020 by Georgia Department of Education. All rights reserved.
English Language Arts (ELA)

Example Item 4
Extended Writing-Response
DOK Level 4: This is a DOK Level 4 item because it requires students to read two passages, synthesize
information, and respond to an extended writing task.
English Language Arts (ELA) Grade 4 Content Domain II: Writing and Language
Genre: Informational
Standards:
ELAGSE4W2. Write informative/explanatory texts to examine a topic and convey ideas and information
clearly.
ELAGSE4L1. Demonstrate command of the conventions of standard English grammar and usage when
writing or speaking.
ELAGSE4L2. Demonstrate command of the conventions of standard English capitalization, punctuation,
and spelling when writing.

This section of the test assesses your skill to comprehend reading passages and use information from the
passages to write an informational essay.
Before you begin writing your essay, you will read two passages.
As you read the passages, think about details you may use in an informational essay comparing and
contrasting the two pilots.
These are the titles of the passages you will read:
1. Charles Lindbergh
2. Amelia Earhart

Georgia Milestones Grade 4 EOG Study/Resource Guide for Students and Parents Page 19 of 152
Copyright © 2020 by Georgia Department of Education. All rights reserved.
English Language Arts (ELA)

Charles Lindbergh
In 1927, Charles Lindbergh became the first person to fly nonstop across the Atlantic Ocean. This
was a famous event in an event-filled life.

Lindbergh was born in 1902. At that time, flying was in its early days. Young Lindbergh found flight
fascinating. He left college to go to flight school. After two years, he went into the U.S. Army. He
became a pilot for the U.S. Army Air Corps.

After the army, Lindbergh flew for the U.S. Postal Service. He flew a mail plane from St. Louis to
Chicago. During this time, he earned his nickname, “Lucky Lindy.” He had to jump out of his plane
four times. He got lucky and lived every time!

In 1919, a man named Raymond Orteig started a contest. He offered $25,000 to the first person
who could fly across the Atlantic Ocean. Lindbergh spent the next eight years getting the right plane.
He named the plane the Spirit of St. Louis. Then, in May 1927, he made his famous flight across the
Atlantic.

Lindbergh received many awards in his life. One was a Pulitzer Prize for a book he wrote about his life.
Lindbergh died at the age of 72.

Page 20 of 152 Georgia Milestones Grade 4 EOG Study/Resource Guide for Students and Parents
Copyright © 2020 by Georgia Department of Education. All rights reserved.
English Language Arts (ELA)

Amelia Earhart
Amelia Earhart was born in 1898 in Kansas. She was a good student. However, she left college at the
age of 19. Soon afterward, Earhart went to an air show in Long Beach, California. It was there that
she took her first airplane ride. It changed her life forever. She started taking flying lessons. Earhart
spent the next couple of years learning all about flying. She even bought her own plane.

Unfortunately, Earhart ran out of money and had to sell her plane. She went back to school for a
while. Then she worked as a teacher and a social worker. In 1927, Charles Lindbergh made his
famous flight across the Atlantic Ocean. People began asking, “Who will be the first woman?” In
1928, Earhart was a passenger on a flight across the Atlantic. She was the first woman to fly across
the Atlantic. She later wrote a book about the experience. But being a passenger wasn’t enough for
Earhart.

In 1935, Earhart became the first person to fly from Hawaii to the U.S. mainland. The U.S.
government gave her a medal for this. In 1937, she decided to try to fly around the world. She made
it to an island in the Pacific Ocean. But then her plane disappeared. She was never found. Earhart
will always be remembered, though. She showed the world what women pilots can do.

Georgia Milestones Grade 4 EOG Study/Resource Guide for Students and Parents Page 21 of 152
Copyright © 2020 by Georgia Department of Education. All rights reserved.
English Language Arts (ELA)

WRITING TASK

Think about the ideas in BOTH passages. Then write an informational essay in your
own words explaining how Charles Lindbergh and Amelia Earhart were similar and
how they were different.
Be sure to use information from BOTH passages in your informational essay.

Writer’s Checklist
Be sure to:
•• Introduce the topic clearly, provide a focus, and organize information in a way that makes
sense.
•• Use information from the two passages so that your essay includes important details.
•• Develop the topic with facts, definitions, details, quotations, or other information and examples
related to the topic.
•• Identify the passages by title or number when using details or facts directly from the passages.
•• Develop your ideas clearly and use your own words, except when quoting directly from the
passages.
•• Use linking words to connect ideas.
•• Use clear language and vocabulary.
•• Provide a conclusion that supports the information presented.
•• Check your work for correct usage, grammar, spelling, capitalization, and punctuation.

Now write your informational essay on your answer document. Refer to the Writer’s Checklist as
you write and proofread your essay.

Page 22 of 152 Georgia Milestones Grade 4 EOG Study/Resource Guide for Students and Parents
Copyright © 2020 by Georgia Department of Education. All rights reserved.
English Language Arts (ELA)

Georgia Milestones Grade 4 EOG Study/Resource Guide for Students and Parents Page 23 of 152
Copyright © 2020 by Georgia Department of Education. All rights reserved.
English Language Arts (ELA)

The following is an example of a seven-point response. See the seven-point, two-trait rubric for a
text-based informational response on pages 87 and 88 to see why this example would earn the
maximum number of points.
Charles Lindbergh and Amelia Earhart had many similarities. They were both pilots at around the same
time. Both left college and studied flying. They were both first at many flying goals, like flying across the
Atlantic Ocean. They both wrote books about what they did.
The two pilots were different in some ways, however. One clear difference is that Lindbergh was a man,
and Earhart was a woman. Also, Lindbergh didn’t have the problems with money that Earhart had. I
think the biggest difference between them, though, was that Lucky Lindy had good luck. He survived four
plane crashes and lived to be 72 years old. But Earhart didn’t have such good luck. At a young age, she
disappeared while trying to fly around the world.
In the end, we will remember both Lindbergh and Earhart for being great pilots.

Page 24 of 152 Georgia Milestones Grade 4 EOG Study/Resource Guide for Students and Parents
Copyright © 2020 by Georgia Department of Education. All rights reserved.
English Language Arts (ELA)

ENGLISH LANGUAGE ARTS (ELA) CONTENT DESCRIPTION AND


ADDITIONAL SAMPLE ITEMS
In this section, you will find information about what to study in order to prepare for the Grade 4 English
Language Arts EOG assessment. This includes main ideas and important vocabulary words. This section
also contains practice questions, with explanations of the correct answers, and activities that you can do
with your classmates or family to prepare for the test.
All example and sample items contained in this guide are the property of the Georgia Department of
Education.

Unit 1: Reading Literary Text

READING PASSAGES: LITERARY TEXT

CONTENT DESCRIPTION
The literary passages in the English Language Arts test are used to identify main ideas and details, cite
evidence, make inferences, determine themes, and understand vocabulary. You may be asked to write
a narrative in response to a prompt based on a literary passage. For more information about narrative
writing, please refer to Unit 3.

Key Ideas and Details


•• Look for ideas and details that tell you what the passage is about.
•• Use these ideas and details when writing or speaking about the passage.
•• Look for themes as you read. Ask yourself, “What is this about?”
•• Think about why the author made the choices he or she made about the characters, setting, and
events in the passage.
•• Use specific details from the passage to help you describe a character’s thoughts, words, or actions.
•• Summarize the important ideas and details after you read.

Craft and Structure


•• Make sure you understand the words and phrases as you read.
•• Think about how the author includes words that allude to significant characters found in mythology.
•• Compare and contrast the structural elements of different passage types, including poems, dramas,
and prose, and be able to explain the similarities and differences.
•• Compare and contrast the point of view in multiple passages, focusing on first- and third-person
narration.

Integration of Knowledge and Ideas


•• Look at the illustration in a passage. Make connections between the illustration and the description,
events, or character traits in the passage.
•• Think about the similarities and differences between themes, topics, and patterns of events in stories,
myths, and traditional literature from different cultures.

Georgia Milestones Grade 4 EOG Study/Resource Guide for Students and Parents Page 25 of 152
Copyright © 2020 by Georgia Department of Education. All rights reserved.
English Language Arts (ELA)

KEY TERMS
Literary text: Passages that are stories, dramas, or poems. (RL)
Inference: To infer means to come to a reasonable conclusion based on evidence found in the passage. By
contrast, an explicit idea or message is stated directly by the author. The author tells the readers exactly
what they need to know. (RL1)
Theme: The theme of a literary passage is its lesson or message. For example, a passage could be about
two friends who like to do things together, and the theme might be the importance of friendship. (RL2,
RL9)
Summarize: To give the main events of a passage in the order in which they happen. (RL2)
Character: A person or thing in a work of literature. Goldilocks is a character in “Goldilocks and the Three
Bears.” (RL3)
Setting: Where and when a literary passage takes place, including the time of day, the season, or the
location. (RL3)
Plot: The events that happen in the beginning, middle, and end of the passage. (RL3)
Mythology: Literary passages about popular beliefs in different cultures. In Greek mythology, the stories of
the Greek gods are very well known and sometimes appear with different names in other cultures, such as
in Roman mythology. (RL4)
Allusion: An allusion is an indirect reference to something. When an author refers to something without
mentioning it explicitly, it is an allusion; for example, The new student is a regular Einstein. In this
sentence, the author is alluding to Albert Einstein, the Nobel Prize-winning scientist and historical figure.
(RL4)
Prose: A form of writing that uses ordinary language. It is the opposite of poetry. (RL5)
Poem: A piece of writing that uses words and phrases chosen for their sound or meaning. These words are
arranged in a specific way on the page and use a number of poetic devices. (RL5)

•• Verse: Writing organized in a rhythmic pattern, as often is the case in poetry. (RL5)
•• Rhythm: The regular, repeated sounds of words in a poem. (RL5)
•• Meter: A rhythm that repeats a basic pattern in a poem. (RL5)

Drama: A story, also known as a play, that is written for people to act out. (RL5)

•• Dialogue: The words that actors are supposed to say or read aloud when they are acting out a
drama. (RL5)
•• Stage directions: The instructions that tell actors what to do in a drama. Stage directions may
also give information about what the stage should look like, what the lighting should be, and what
sounds should be heard. For example, stage directions tell actors when to enter the stage, when to
exit, or what specific actions they should do while speaking. (RL5)
•• Cast of characters: The list of different parts actors can play in the drama. Usually, each character
is played by a different actor. (RL5)

Point of view: The perspective from which a story is told. The point of view depends upon who the narrator
is and how much he or she knows. The point of view could be first person (I went to the store), second
person (You went to the store), or third person (He went to the store). The point of view used by the author
can have a big influence on his or her passage. (RL6)

Page 26 of 152 Georgia Milestones Grade 4 EOG Study/Resource Guide for Students and Parents
Copyright © 2020 by Georgia Department of Education. All rights reserved.
English Language Arts (ELA)

Compare vs. contrast: Though similar, comparing is analyzing two things, such as characters or themes, in
relation to each other, while contrasting is specifically analyzing the differences between two things, such
as two different characters or themes. (RL6, RL9)
Genre: A category of passages, such as fiction and nonfiction. Each genre has a particular style, form, and
content. (RL9)
Pattern of events: What happens in the story. This is often referred to as the plot or the sequence of
events that make up the story. For example, a quest is a pattern of events in which a character searches
for something. (RL9)
Traditional literature: Stories that were first told verbally but were written down much later. These stories
do not happen in a specific time or place. Instead, they share values or common themes. For example,
the stories of Cinderella, Sleeping Beauty, and the Three Little Pigs all had their beginnings in traditional
literature. (RL9)

Important Tips
"" Use details to support ideas and to answer what you know and how you know it.
"" When responding to an item, try to answer the question being asked before you read the answer
choices.
"" Look for familiar prefixes, suffixes, and word roots to help you decide the meaning of an unknown word.

Georgia Milestones Grade 4 EOG Study/Resource Guide for Students and Parents Page 27 of 152
Copyright © 2020 by Georgia Department of Education. All rights reserved.
English Language Arts (ELA)

Sample Items 1–8

Read the story and answer questions 1 through 8.

The Piano
Greta did not like change. She didn’t like it when they changed the design on the wrapper of her
favorite kind of ice cream (strawberry mango). She didn’t like it when her father shaved his beard,
because it made him look too young. And she didn’t like it when she came home to find this . . .
thing in the living room. It was old and brown and heavy, and it practically took up a whole wall. It had
yellow teeth, and it made noise. It was a piano.

When her older brother Richard started banging on the instrument with his elbows, Greta ran to her
room and closed the door. That’s when she noticed something even worse: the toy chest was now
in the middle of her room. This was the toy chest that had stood against the wall in the living room
forever.

“Don’t you like our new family member?” Greta’s mother asked from outside the door.

Greta opened the door and blurted, “Where did you get that thing?”

“The Kleins are moving out, and they didn’t want to move it.”

“The Kleins are moving?” That meant more change. The Kleins had always lived in the apartment
down the hall. It was all too much.

“Yes, and it will all be fine,” said Greta’s mother, and she walked back toward the living room.

Greta closed the door, flopped onto her bed face-down, and did what she always did when she didn’t
like what was going on: she fell asleep.

In her dream, Greta was floating on a raft in the middle of a river. The breeze stirred the water and
made the most beautiful sound. The sound rose and fell and became louder when the wind became
stronger.

Greta woke up and opened her eyes, but the sound continued. She got up and stumbled into the
living room, where her mother sat at the piano. The sound was pouring out of her mother’s fingers,
but she was looking straight ahead with her head cocked slightly to the right. Then she stopped
playing and remained still, in a different world.

“Come and sit here,” Greta’s mother said as she scooted over and patted the bench next to her.

“I didn’t know . . . ,” Greta began.

“Of course you didn’t know, because I never told you I could play. I started when I was about five
years old.”

“Why did you stop?”

Page 28 of 152 Georgia Milestones Grade 4 EOG Study/Resource Guide for Students and Parents
Copyright © 2020 by Georgia Department of Education. All rights reserved.
English Language Arts (ELA)

“I didn’t really stop. I guess I kind of drifted away from it. When I moved out of my parents’ house, I
left the piano behind, and I never got another one.”

Greta stared at her mother’s face, which held a half smile. “You never should have stopped,” said
Greta.

“You might be right,” Greta’s mother said, and she stole a quick look at her daughter.

Greta felt like her mother had just told her a secret, and a bubble of warmth rose inside her.

“Will you play some more?” she asked.

Item 1
Selected-Response
What is the MOST LIKELY reason Greta’s mother invites Greta to sit on the piano bench with her?

A. She would like to show Greta how to play the piano.


B. She knows Greta is upset and is trying to comfort her.
C. She would like to explain to Greta how she learned to play the piano.
D. She knows Greta is nervous about having new neighbors.

Item 2
Selected-Response
Which explanation BEST describes the meaning of the phrase drifted away from it as it is used in
these sentences from the story?

“I didn’t really stop. I guess I kind of drifted away from it. When I moved out of my parents’
house, I left the piano behind, and I never got another one.”

A. Greta’s mother stopped enjoying music.


B. Greta’s mother felt sad about playing music.
C. Greta’s mother stopped playing the piano bit by bit.
D. Greta’s mother suddenly finished listening to a song.

Georgia Milestones Grade 4 EOG Study/Resource Guide for Students and Parents Page 29 of 152
Copyright © 2020 by Georgia Department of Education. All rights reserved.
English Language Arts (ELA)

Item 3
Selected-Response
At the end of the story, Greta asks her mother, “Will you play some more?”

Based on the story, why does Greta MOST LIKELY ask this question?

A. She wants her neighbors to hear her mother playing the piano.
B. She worries that her mother does not like playing the piano.
C. She hopes her mother can become better at playing the piano.
D. She enjoys hearing her mother playing the piano.

Item 4
Selected-Response
Why does the author use dialogue in the scene at the piano?

A. to show how Greta’s feelings are changing


B. to show why Greta’s mother is upset
C. to show Greta’s excitement about having new neighbors
D. to show that Greta’s mother worries about her daughter

Item 5
Selected-Response
Which sentence correctly states the point of view in the story?

A. The story is told by a narrator who provides Greta’s point of view.


B. The story is told by a narrator who provides Greta’s mother’s point of view.
C. The story is told by a narrator who provides Richard’s point of view.
D. The story is told by a narrator who provides many family members’ points of view.

Page 30 of 152 Georgia Milestones Grade 4 EOG Study/Resource Guide for Students and Parents
Copyright © 2020 by Georgia Department of Education. All rights reserved.
English Language Arts (ELA)

Item 6
Evidence-Based Selected-Response Technology-Enhanced
This question has two parts. Answer Part A, and then answer Part B.

Part A
Which word BEST describes Greta’s mother?

A. generous
B. humorous
C. talented
D. forgiving

Part B
Which sentence from the story BEST supports the answer in Part A?

A. “Yes, and it will all be fine,” said Greta’s mother, and she walked back toward the living room.
B. The sound was pouring out of her mother’s fingers, but she was looking straight ahead with her head
cocked slightly to the right.
C. “You might be right,” Greta’s mother said, and she stole a quick look at her daughter.
D. Greta felt like her mother had just told her a secret, and a bubble of warmth rose inside her.

Georgia Milestones Grade 4 EOG Study/Resource Guide for Students and Parents Page 31 of 152
Copyright © 2020 by Georgia Department of Education. All rights reserved.
English Language Arts (ELA)

Item 7
Drag-and-Drop Technology-Enhanced

ÂÂ
Due to the size of the response area, this item has a “Click To Respond” button on the screen.
Clicking this button will bring up the response area at full size.

Go on to the next page to finish item 7.

Page 32 of 152 Georgia Milestones Grade 4 EOG Study/Resource Guide for Students and Parents
Copyright © 2020 by Georgia Department of Education. All rights reserved.
English Language Arts (ELA)

Item 7. Continued.

ÂÂ
Use a mouse, touchpad, or touchscreen to move the descriptions below the table into the boxes in
the table.

Georgia Milestones Grade 4 EOG Study/Resource Guide for Students and Parents Page 33 of 152
Copyright © 2020 by Georgia Department of Education. All rights reserved.
English Language Arts (ELA)

Item 8
Extended Constructed-Response
Write an ending to the story that starts with Greta’s mother saying to Greta, “Now tell me why you
were so upset when you saw the piano.”

Be sure that your story ending flows naturally from the rest of the story. Use dialogue and
descriptions in your story.

Narrative Writer’s Checklist


Be sure to:
•• Develop a real or imagined experience.
•• Establish a situation and introduce a narrator and/or characters.
•• Organize events in order.
◦◦ Use transitional words and phrases to sequence the events.
•• Use dialogue and/or description to:
◦◦ develop events.
◦◦ show how characters respond to situations.
•• Use concrete words, phrases, and sensory details to describe the events.
•• Include a conclusion.
•• Use ideas and/or details from the passage(s).
•• Check your work for correct usage, grammar, spelling, capitalization, and punctuation.

Now write your narrative on your answer document. Refer to the Writer’s Checklist as you write
and proofread your narrative.

Page 34 of 152 Georgia Milestones Grade 4 EOG Study/Resource Guide for Students and Parents
Copyright © 2020 by Georgia Department of Education. All rights reserved.
English Language Arts (ELA)

Georgia Milestones Grade 4 EOG Study/Resource Guide for Students and Parents Page 35 of 152
Copyright © 2020 by Georgia Department of Education. All rights reserved.
English Language Arts (ELA)

Unit 2: Reading Informational Text

READING PASSAGES: INFORMATIONAL TEXT

CONTENT DESCRIPTION
The informational passages in the English Language Arts test can be used to determine main ideas,
write objective summaries, analyze ideas, and provide supporting text evidence. You may be asked to
write a narrative in response to a prompt based on an informational passage. For more information about
narrative writing, please refer to Unit 3.

Key Ideas and Details


•• Think about the passage and try to paraphrase and summarize as you read.
•• Look for ideas and details that tell you what the passage is about.
•• Use these ideas and details when writing or speaking about the passage.
•• Think about how key details, examples, and inferences help you determine the main idea in the
passage.
•• Use specific details from the passage when explaining events, procedures, ideas, and concepts.

Craft and Structure


•• Make sure you understand the words in the passage.
•• Understand common organizational structures, such as chronological order, cause and effect, and
comparison, and recognize when they are used in a passage.
•• Understand how a firsthand account differs from a secondhand account when reading two passages
about the same event or topic.

Integration of Knowledge and Ideas


•• Understand how graphic features, such as charts, graphs, maps, timelines, and illustrations, are used
to help the reader understand the passage.
•• Think about how the author uses reasons and evidence to support particular points in the passage.
•• Integrate information from two passages on the same topic in order to show an understanding of the
topic.

Page 36 of 152 Georgia Milestones Grade 4 EOG Study/Resource Guide for Students and Parents
Copyright © 2020 by Georgia Department of Education. All rights reserved.
English Language Arts (ELA)

KEY TERMS
Informational text: Passages that explain or inform. (RI)
Inference: To infer means to come to a reasonable conclusion based on evidence found in the passage. By
contrast, an explicit idea or message is stated directly by the author. The author tells the readers exactly
what they need to know. (RI1)
Main idea: The most important idea that the author is trying to say. (RI2)
Key details: The facts and ideas that support the main idea of a passage. (RI2)
Summary: A summary contains the most important points from a passage but does not give all the details.
(RI2)
Organization: Organization refers to the way in which a piece of writing is structured. Each sentence,
paragraph, or chapter fits into the overall structure of a text and contributes to the development of ideas.
Organizational structures include chronological order, cause and effect, compare and contrast, and
problem and solution. (RI5)

•• Chronological order: The order in which a series of events happens. A passage that is arranged in
order of time from the beginning to the end is in chronological order. (RI5)
•• Compare and contrast: Compare and contrast analyzes the relationships between ideas in a text.
Comparing analyzes the similarities, while contrasting analyzes the differences. (RI5, RI6)
•• Cause and effect: This is a relationship where one thing causes another thing to happen.
(RI3, RI5)
•• Problem and solution: Text that is organized by problem and solution identifies a problem and
proposes one or more solutions. An author may use problem and solution to try to persuade
readers about a certain topic or course of action. (RI5)

Firsthand account: A description of events written or told by someone who was actually there. If your
friend tells you she fell and hurt her knee, it is a firsthand account. (RI6)
Secondhand account: A description of events written or told by someone who was not actually there but
who got the story from another source. If your friend tells you that your other friend fell and hurt her knee,
but the friend who is telling you didn’t see the fall happen, it is a secondhand account. (RI6)
Evidence: Something that proves the truth of something else. Informational texts may contain evidence in
the form of key words, illustrations, maps, or photographs to prove that the information is correct. (RI7)
Author’s purpose: The author has a specific reason or purpose for writing the passage. Often the author’s
purpose is not directly stated. (RI8)

Important Tips
"" Try to read the questions about an informational passage before you read the passage so that you
know what to look out for.
"" Use evidence from a passage to help explain what is being said.
"" Use facts and details to support ideas and answer what you know and how you know it.

Georgia Milestones Grade 4 EOG Study/Resource Guide for Students and Parents Page 37 of 152
Copyright © 2020 by Georgia Department of Education. All rights reserved.
English Language Arts (ELA)

Sample Items 9–11

Read the article and answer questions 9 through 11.

The Statue of Liberty


The Statue of Liberty is one of the world’s most famous statues. Lady Liberty stands with a torch in
her hand. She has been welcoming ships into New York City’s harbor since 1886. Many people know
that the statue was a gift from France to the United States. But the story is not that simple.

The idea to make a statue as a gift began in France. An artist named Frédéric-Auguste Bartholdi
wanted to build the statue, but he needed the money to do it. He formed a group in France. They
decided to raise money in France to pay for the copper statue. However, Lady Liberty needed a base
to stand on. That money was to be raised in the United States.

Many people in France gave money for the statue. Even schoolchildren contributed. A copper
company gave Bartholdi all the copper he needed.

Bartholdi made the right arm and hand of the statue. It was put on display in Philadelphia and New
York City. People became excited about the statue. Americans began to give money to complete it.
But there still wasn’t enough money for the base.

Then Bartholdi came up with a good idea. In New York he spread the word that the statue might go
to Boston or another city. The idea worked. New Yorkers didn’t want to be left out. The people of New
York donated more money. Then Bartholdi could complete the base. Now Lady Liberty stands in New
York Harbor. The people of France and the United States worked together. Like most great works, it
took a long time. It also took a lot more work than most people think.

Page 38 of 152 Georgia Milestones Grade 4 EOG Study/Resource Guide for Students and Parents
Copyright © 2020 by Georgia Department of Education. All rights reserved.
English Language Arts (ELA)

Item 9
Selected-Response
Which sentence from the article explains why enough money was finally raised for the base?

A. That money was to be raised in the United States.


B. People became excited about the statue.
C. Americans began to give money to complete it.
D. New Yorkers didn’t want to be left out.

Item 10
Selected-Response
What does the phrase spread the word mean in this sentence from the article?

In New York he spread the word that the statue might go to Boston or another city.

A. told a lie
B. kept a secret
C. wrote a large sign
D. told a lot of people

Item 11
Selected-Response
With which statement would the author MOST LIKELY agree?

A. People often do not know the real story behind events.


B. No one knows how the Statue of Liberty was really built.
C. The money for the Statue of Liberty came only from France.
D. Americans do not care enough about their nation’s history.

Georgia Milestones Grade 4 EOG Study/Resource Guide for Students and Parents Page 39 of 152
Copyright © 2020 by Georgia Department of Education. All rights reserved.
English Language Arts (ELA)

Unit 3: Writing

CONTENT DESCRIPTION
In this unit, you will be reading passages that are similar to passages you may read in the Georgia
Milestones End-of-Grade assessment. You will use the passages as sources of support for opinion and
informational/explanatory essays and as jumping-off points for narrative writing.
Some informational passages will help you develop opinions and support your point of view on a topic
in an opinion essay. In your writing, you will use evidence to develop and support your opinion. Other
informational passages will help you develop an informational/explanatory essay. In your writing, you will
state ideas, summarize information, and use details from more than one source to develop and support
your ideas.
You will also write a narrative in response to a prompt based on a literary or informational passage or
a paired passage set you have read. A paired passage set may consist of two literary passages, two
informational passages, or one of each passage type. Narrative prompts will vary depending on the
passage(s) you are shown. For example, you may be asked to write a new beginning or ending to a literary
story, write an original story based on information from an informational text, or rewrite a scene from a
specific character’s point of view. In your writing, you will use narrative techniques to develop the reader’s
understanding of a real or imagined experience.
There will also be writing standalone items that assess your revision skills and your understanding of
opinion, informational/explanatory, and narrative writing. For example, you may be asked to answer a
selected-response question that focuses on introducing a topic in an informational text. In addition, there
will be writing standalone items that assess your planning and research skills.

Writing Types and Purposes


Opinion Essay
•• An opinion essay states an opinion or agrees or disagrees with a point of view.
•• Some common opinion words are “agree,” “disagree,” “for,” or “against.”
•• When you state your opinion, you need to support it with reasons, examples, and evidence.
Informational/Explanatory Essay
•• An informational/explanatory essay states ideas and information clearly and accurately.
•• When you develop your topic, use facts, definitions, and details related to your topic.
Narrative
•• A narrative develops a real or imagined experience or event.
•• When you develop your narrative, use narrative techniques, descriptive details, and event sequence.
Production and Distribution of Writing
•• Use the writing process to develop opinion essays, informational/explanatory essays, and narratives.
•• Produce writing with an organization and style that fit the task, purpose, and audience.
•• Strengthen your writing by reviewing or revising, if needed.
Opinion Essay
•• Introduce a topic or text clearly by stating your opinion and create an organizational structure in which
related ideas are grouped to support your opinion.
•• Develop your opinion by providing reasons that are supported by facts and details.

Page 40 of 152 Georgia Milestones Grade 4 EOG Study/Resource Guide for Students and Parents
Copyright © 2020 by Georgia Department of Education. All rights reserved.
English Language Arts (ELA)

•• Use linking words and phrases to connect opinions and reasons.


•• Provide a concluding statement related to the opinion you present.
Informational/Explanatory Essay
•• Introduce a topic clearly and group related information about the topic in paragraphs and sections.
•• Develop your topic with facts, definitions, concrete details, quotations, and other information and
examples related to the topic.
•• Use linking words and phrases to connect ideas within categories of information.
•• Use precise language to inform about or explain the topic.
•• Provide a concluding statement or section related to the information or explanation.
Narrative
•• Include a situation and introduce a narrator and/or characters.
•• Organize an event sequence that unfolds naturally.
•• Develop your narrative by using dialogue and/or descriptions of actions, thoughts, and feelings to
develop events or show how characters respond to situations.
•• Use a variety of transitional words and phrases to sequence the events in the narrative.
•• Use concrete words and phrases and sensory details to clearly describe experiences and events.
•• Include a conclusion.
Audience, Purpose, and Voice
•• As you write, remember who your audience will be.
•• Remember, you are writing for a purpose—think about what you are writing and why.
•• As you write opinion or informational/explanatory essays, reveal your writing voice by using language
that matches the content, connects with your intended readers, and reveals your personality and
writing style.
•• As you write your narrative, reveal your writing voice by choosing a narrator and point of view that allow
your readers to experience the story and relate to the characters in a meaningful way.
Research to Build and Present Knowledge
•• Conduct a short research project that builds knowledge through investigation of different aspects of a
topic.
•• Gather information from different types of sources, including print and digital sources.
•• Take brief notes on the sources and sort the information about the topic into categories.
•• Provide a list of sources you used for your research.
Range of Writing
•• Write routinely over extended time frames (time for research, reflection, and revision) and shorter time
frames (a single sitting or a day or two) for a range of tasks, purposes, and audiences.
Scoring Rubrics
•• A narrative scoring rubric can be found beginning on page 85. An informational/explanatory scoring
rubric can be found beginning on page 87. An opinion scoring rubric can be found beginning on
page 89. You may find it helpful to read and discuss these rubrics with a parent or another adult.
•• The rubrics are important to understand because they show you what is needed to produce a strong
piece of opinion, informational/explanatory, or narrative writing.
•• Opinion, informational/explanatory, and narrative writing on the EOG assessment will be scored using
these rubrics.
Georgia Milestones Grade 4 EOG Study/Resource Guide for Students and Parents Page 41 of 152
Copyright © 2020 by Georgia Department of Education. All rights reserved.
English Language Arts (ELA)

KEY TERMS
Opinion text: An opinion text states an opinion or agrees or disagrees with a point of view. (W1)
Point of view: The opinion or perspective of the writer on a specific topic. (W1)
Topic: What a piece of writing is about. When writing your opinion, choose topics about which you have
strong feelings and a lot to say. (W1a, W2a)
Introduction: The beginning of a piece of writing. The introduction should let readers know what they will
be reading about, and it should set up the main idea, or thesis, of the writing. (W1a, W2a)
Organization: The way in which a piece of writing is structured. Similar ideas and illustrations should be
grouped together and the order of the information should make sense. Each sentence, paragraph, or text
feature fits into the overall structure of a passage and contributes to the development of ideas. Writers
structure their texts to match their purpose and audience. (W1a, W4)
Reasons: Details that support your opinion in a piece of writing. (W1b)
Fact and opinion: A fact is a statement that can be proven. An opinion is a statement that cannot be
proven because it states a writer’s belief or judgment about something. Deciding whether a statement is a
fact often comes down to a single question: “Can you prove it?” If you can prove a statement, then it is a
fact. If not, it’s an opinion. (W1b)
Linking words and phrases: Words or groups of words that link one idea to the next. Writing should not
jump from one idea to the next without transitions that guide the reader along. Examples of linking words
include also, another, and, more, because, therefore, since, and but. Examples of linking phrases are to
begin, on the other hand, for example, and in conclusion. (W1c, W2d)
Concluding statement: The end of a piece of writing. The concluding statement should sum up the main
purpose of the writing and provide an overall takeaway for the reader. (W1d, W2e)
Informational/explanatory texts: A form of writing that informs the reader or explains something. (W2)
Formatting: The way in which a piece of writing is organized. For example, a writer can use headings and
subheadings to organize the writing and present the information in a clear way. (W2a)
Precise language: Good writers choose their words carefully. Specific and vivid words and phrases
describe or explain and make meaning clear. The sentence A bird was on the ground is very general and
does not use precise language. However, that sentence could be rewritten using more specific nouns and
verbs: A robin landed in the grassy field. (W2d)
Narrative: A narrative is a real or imaginary story. It may be about a situation, a single moment in time,
or a series of related events and experiences. Narratives may be about a single moment in time but focus
on how one character thinks and feels about it. Narratives may be about a single moment in time but
focus on how several different characters think and feel about it. Narratives may also be about a series of
related events and experiences and how a character feels about them. Good writers order the thoughts,
feelings, events, and experiences in a way that makes sense to the reader. (W3)
Experience: An experience is an event or series of events that happen to or are seen by a character. In
a narrative, characters learn, grow, or find meaning by going through an experience. An experience in a
narrative may be described as a single event (Mary’s team won the soccer game) or as a series of events
(Mary kicked the ball toward the goal. The goalie dived but missed the ball. The ball landed inside the goal.
Mary had scored the winning point. Her team won the game!). A character may be part of an experience
(Mary kicked the soccer ball into the goal). A character may also watch or observe an experience without
being part of it (Mary watched as her two older brothers played against each other on different soccer
teams). A narrative may include more than one experience. (W3, W3a, W3b)

Page 42 of 152 Georgia Milestones Grade 4 EOG Study/Resource Guide for Students and Parents
Copyright © 2020 by Georgia Department of Education. All rights reserved.
English Language Arts (ELA)

Event: An event is a single thing that happens to a character or that a character sees. For example, John
caught the soccer ball is an event. It is a single thing that happened to the character John. (W3, W3a,
W3b)
Orient the reader: Readers need to understand who is telling the story. Readers also need to know the
setting, characters, and conflict or challenge of the story. Good writers share this information early in a
narrative so that readers can follow the events, thoughts, and experiences of a story and understand the
story’s meaning or message. Sharing this information in a meaningful way is called orienting the reader.
(W3a)
•• Introduction: The introduction is the first few paragraphs of a narrative that orient the reader to the
story. Good writers create an introduction that interests the reader and shares just enough information
to keep the reader reading and learning more. There is no one right way to write an introduction.
Introductions may include dialogue, a description of the setting, an introduction of the narrator, a
description of a character, an explanation of the situation, or any combination of these. Good writers
create a unique introduction that best fits their story and its meaning. (W3a)
•• Narrator: The narrator is the person the writer chooses to tell a story. The narrator may be a character
in the story. The narrator may also record the characters’ actions, words, and thoughts but not be a
character in the story. (W3a)
•• Characters: Characters are persons, things, or beings in stories. The characters may be real or
imaginary. The details you share about characters—the way they think, talk, and act—help the reader
understand the characters’ personalities. (W3a, W3b)
Sequence: Sequence is the order of events in a narrative. Often, the events in a story are told in the exact
order they happened. Sometimes, a story is told out of order to create surprise or to help the reader learn
more about the characters and how they think and behave. (W3, W3a)
Narrative techniques: Narrative techniques are the tools writers use to create interesting experiences,
events, and characters in a story. (W3, W3b)

•• Dialogue: Writers use dialogue to show the reader the exact words the characters are saying. Dialogue
usually has quotation marks around it. Each time a new character speaks, a new paragraph begins.
The reader learns about characters from the way they speak or respond to a situation. Dialogue can
also move the action forward in a story or cause a character to decide something. (W3b)
•• Description: Good writers use description to help the reader imagine the characters, settings, and
events. Description helps make it feel like the reader is living the events of the story, both seeing what
a character sees and feeling what the character feels. This sentence does not have good description:
The kids at my new school were friendly. These sentences use description to help the reader see and
feel what the character experiences: I stepped into the classroom. I worried that I would not make new
friends in my class. After the teacher introduced me, she asked me to tell the class where I was from.
“I moved to Georgia from India,” I said. “This is my first time in the United States.” Everyone in the
class smiled at me with shining eyes. “Welcome to our class,” a girl in the front row said. “Would you
like to sit with me at lunch today?” (W3b)
Transitional words and phrases: The reader needs clues in a story to know how time is passing and
how events are ordered. Transitional words and phrases link one idea to the next and help the reader
understand how time is passing in the story. Transitional words and phrases also make clear the order in
which events happen. Examples of transitional words are first, next, before, during, and finally. Examples of
transitional phrases are after that, in the beginning, it started when, and the next day. (W3c)
Concrete words and phrases: Concrete words and phrases refer to physical things that can be seen,
heard, smelled, tasted, or touched. Examples are sidewalk, pizza, plate, tree, and puppy. Good writers use
concrete details to help the reader understand exactly what the writer means and picture a scene clearly.

Georgia Milestones Grade 4 EOG Study/Resource Guide for Students and Parents Page 43 of 152
Copyright © 2020 by Georgia Department of Education. All rights reserved.
English Language Arts (ELA)

The sentence Ani wanted a friend is not concrete because the word friend means different things to
different people. This sentence uses concrete words to help the reader understand exactly what Ani wants:
Ani wanted a striped furry kitten that would rub against his pant legs, chase toy balls, and curl up next to
him in bed at night. (W3d)
Sensory details: Sensory details describe concrete words and phrases in a way that allows the reader
to experience the way things look, sound, smell, taste, or feel through imagination. Good writers share
sensory details to help the reader imagine what it would be like to live the experience in the story. For
example, the sentence The hot spring stank does not help the reader imagine what the hot spring smelled
like. The sentence The hot spring smelled like rotten eggs that had blown up in the microwave helps the
reader better imagine the smell. (W3d)
Conclusion: Every story needs to have an end. The end can be anywhere the writer chooses to stop
writing. But the reader needs to feel like the story is over. Good writers create this feeling of ending with
a conclusion. In the conclusion, the events of the story stop, and the reader understands one or more of
these ideas: what the story meant, what characters learned, how characters felt about the experience, how
characters changed, and what the reader can learn from the story. (W3e)
Audience: The people who will be reading the piece of writing. Writers should keep their audience in mind
and adjust their ideas and vocabulary so that they can be best understood. (W4)
Purpose: The writer’s reason for writing his or her essay or article. All writing has a purpose, whether it is
to persuade, inform, explain, or entertain. (W4)
Revision: The process of editing and rewriting a piece of writing. All good writing requires a lot of revision
in order to catch mistakes and make ideas clearer. (W5)
Research: Gathering information in order to learn more about a topic. (W7)
Source: A book, article, website, person, or piece of media that contains information. (W8)
Evidence: Something that proves the truth of something else. Informational texts may include facts,
opinions of experts, quotes, statistics, and definitions that can be used as evidence. In literary text, the
characters’ thoughts, words, or actions may be used as evidence. (W9)

Important Tips
Opinion and Informational/Explanatory Essays
"" Organize your writing by using an organizational structure in which related ideas are grouped together.
"" In your opinion essay, be sure to develop your opinion with reasons that are supported by facts and
details. In your informational/explanatory essay, be sure to develop your informational topic with
details, such as facts, definitions, quotations, or other information that supports your topic.
"" Make sure your writing has a concluding statement related to the opinion or information presented.
Narrative
"" Organize thoughts, ideas, or events in a sequence.
"" Use dialogue and description to develop events and to show how characters respond to situations.
"" Make sure your narrative has a conclusion.
Opinion, Informational/Explanatory, and Narrative Writing
"" Strengthen your writing by planning, revising, editing, rewriting, or trying a new approach.
"" Use the writer’s checklist before, during, and after writing to make sure you are meeting the
criteria.

Page 44 of 152 Georgia Milestones Grade 4 EOG Study/Resource Guide for Students and Parents
Copyright © 2020 by Georgia Department of Education. All rights reserved.
English Language Arts (ELA)

SAMPLE ITEMS
The practice writing items for this unit include an extended writing-response item, an extended
constructed-response item, and writing standalone items. There are also sample reading comprehension
items connected to the passages you will read in this unit. You will have sample selected-response,
evidence-based selected-response, and/or constructed-response items in this section. In the actual
assessment, there is often a mix of reading comprehension and extended constructed-response and/or
extended writing-response items connected to one passage or passage set.
Sample Items 12–15
Extended Writing-Response (Opinion or Informational/Explanatory Essay)
In Section 1 of the Georgia Milestones End-of-Grade assessment, you will be asked to comprehend a pair
of informational passages and use information from the passages to write an opinion or informational/
explanatory essay. In the End-of-Grade assessment, the task will include the following items:
1. Three selected-response (multiple-choice) questions (three in this example)
2. A constructed-response question (no constructed-response question in this example)
3. An extended writing-response question (one in this example)

The instructions for the extended writing prompt are in the same form as those that appear on the Georgia
Milestones assessment. In the actual assessment, you will receive either an opinion or an informational/
explanatory writing task. The sample provided in this resource is an example of an opinion writing task.
This section of the test assesses your skill to comprehend reading passages and use information from the
passages to write an opinion essay.
Before you begin writing your essay, you will read two passages and answer three multiple-choice questions
about what you have read.
As you read the passages, think about details you may use in an opinion essay about weekend homework.
These are the titles of the passages you will read:
1. Homework on the Weekend
2. Weekends Are for Fun

Georgia Milestones Grade 4 EOG Study/Resource Guide for Students and Parents Page 45 of 152
Copyright © 2020 by Georgia Department of Education. All rights reserved.
English Language Arts (ELA)

Homework on the Weekend


Homework on the weekend is an important part of our education. First of all, we go to school to
prepare for the real world. In the real world, most people work long hours. They may work nights and
on the weekends. Sometimes, they don’t have a choice about weekend work. Learning is the same
way. It doesn’t happen just during the school week. Doing homework on the weekend is another way
to help you learn.

It’s true that there is no school on the weekends. Many students look at the weekend as a time to
play or to do other fun activities. No one is saying you need to stay in and do a lot of homework. You
need some time for fun. But an hour or so of homework should be fine. There is plenty of time over
the weekend to get it done and go have fun as well.

Finally, many students want to go to a college or university. Students do whatever is necessary to


help them get into college, even if that means doing homework on the weekend.

Page 46 of 152 Georgia Milestones Grade 4 EOG Study/Resource Guide for Students and Parents
Copyright © 2020 by Georgia Department of Education. All rights reserved.
English Language Arts (ELA)

Weekends Are for Fun


Homework on the weekends is more harmful than helpful. One university study explored the effects
of homework. The study leaders asked, Does homework help students do better in school? Homework
had very little effect on younger kids especially. If homework isn’t helping us, why have it on
weekends?

Homework can actually harm students. Young people need their weekends. They should forget about
school. They should just be kids. Weekend homework is stressful for kids. It ruins their time off.

On weekends kids should spend time with their families. Sports and hobbies are also important.
What happens if kids can’t do these things? They are tired and unhappy on Mondays. Tired, unhappy
students don’t perform well. Therefore, teachers should not give homework on the weekends.

Georgia Milestones Grade 4 EOG Study/Resource Guide for Students and Parents Page 47 of 152
Copyright © 2020 by Georgia Department of Education. All rights reserved.
English Language Arts (ELA)

Item 12
Selected-Response
Why does the author of “Homework on the Weekend” MOST LIKELY mention college?

A. College students often work jobs on weekends.


B. Students in college have to study on weekends.
C. Weekend homework might help students get into college.
D. College is more like the real world than elementary school is.

Item 13
Selected-Response
Which sentence from “Weekends Are for Fun” explains why students would do better in school if they
had no homework on weekends?

A. Homework had very little effect on younger kids especially.


B. If homework isn’t helping us, why have it on weekends?
C. On weekends kids should spend time with their families.
D. Tired, unhappy students don’t perform well.

Item 14
Selected-Response
Which sentence describes something the reader learns from reading BOTH passages?

A. Homework is important for young kids.


B. Students should think about their futures.
C. There should be no homework on weekends.
D. It is important to have time to play on the weekends.

Page 48 of 152 Georgia Milestones Grade 4 EOG Study/Resource Guide for Students and Parents
Copyright © 2020 by Georgia Department of Education. All rights reserved.
English Language Arts (ELA)

Item 15
Extended Writing-Response

WRITING TASK

Both passages discuss the idea of giving students homework on weekends.


Think about the ideas in BOTH passages. Then write an opinion essay about whether
students should be given homework on the weekend.
Be sure to use information from BOTH passages in your opinion essay.

Writer’s Checklist
Be sure to:
•• Introduce your opinion.
•• Support your opinion with reasons and details from the passages.
•• Give your reasons and details in a clear order.
•• Develop your ideas clearly and use your own words, except when quoting directly from the
passages.
•• Identify the passages by title or number when using details or facts directly from the passages.
•• Use linking words, phrases, and clauses to connect reasons.
•• Use clear language and vocabulary.
•• Have a strong conclusion that supports your opinion.
•• Check your work for correct usage, grammar, spelling, capitalization, and punctuation.

Now write your opinion essay on your answer document. Refer to the Writer’s Checklist as you write
and proofread your essay.

Georgia Milestones Grade 4 EOG Study/Resource Guide for Students and Parents Page 49 of 152
Copyright © 2020 by Georgia Department of Education. All rights reserved.
English Language Arts (ELA)

Page 50 of 152 Georgia Milestones Grade 4 EOG Study/Resource Guide for Students and Parents
Copyright © 2020 by Georgia Department of Education. All rights reserved.
English Language Arts (ELA)

Sample Items 16–19

Extended Constructed-Response (Narrative)


On the Georgia Milestones End-of-Grade assessment, you will write a narrative in response to a prompt
based on a literary or informational passage or a paired passage set you have read. In the actual
assessment, you will also respond to reading comprehension questions before writing your narrative.
Narrative prompts will vary depending on passage type. The sample provided in this resource is an example
of a narrative prompt based on an informational paired passage set.
Read the letters and answer questions 16 through 19.

Pen Pals: A Letter to Mia


Dear Mia,
Hola! My name is Franco Reyes. I live on a farm with my parents, my younger sister, and my older
brother. We speak Spanish, but I am writing to you in English.
We live just outside of a town called Loncotraro, in southern Chile. The country of Chile is in South
America. The Pacific Ocean is on our western border. The country of Argentina is to our east.
Our school year begins in March, ends in December, and we have two weeks off in July for winter
break. My teacher says this is different where you live. When do you go to school?
Each morning, I get up at 6:30 A.M. and put on my school uniform. Then I eat breakfast with my
family. Usually, I eat toast and butter with a glass of milk. What do you like to eat for breakfast?
I ride the city bus to school with my siblings and cousins. Sometimes Aunt Catalina drives us to the
bus stop, but usually we walk. Our bus ride into town takes 20 minutes. Then we walk through town to
get to school. How do you get to school?
School begins at 8:15 A.M., and my first class is language arts. Recently, we’ve been writing short
stories. I enjoy writing stories about space and sharing them with my friends. My class also studies
math and science in the morning. In science, we are studying planets.
In the afternoon, we have physical education, music, and art classes. My favorite sport to play in
physical education is soccer. My whole family loves to play and watch soccer. We also take English
lessons at school. Some people think learning a new language is difficult, but I think it is easy. My
mom speaks English, so she practices with me. She works as a guide taking tourists hiking. Do you
study another language?
At 4:00 P.M., my siblings, cousins, and I ride the city bus home. At home, we play outside together
and ride our horses. At dinner, we often eat sandwiches with either honey, butter, avocado, or ham on
them. After dinner, I do my homework. At 8:30 P.M., I shower, brush my teeth, and read a chapter of
a book before I fall asleep.
Please write back so that I can learn about what you do each day!
Your new friend,
Franco Reyes

Georgia Milestones Grade 4 EOG Study/Resource Guide for Students and Parents Page 51 of 152
Copyright © 2020 by Georgia Department of Education. All rights reserved.
English Language Arts (ELA)

Pen Pals: A Letter to Franco


Dear Franco,

I was excited to get your letter! We have learned a lot about Chile in school. It is interesting to meet
someone who lives there.

I live with my mom in an apartment building in New York City, which is the biggest city in the country!
New York City is on the east coast of the United States. We have a big harbor in New York City that is
on the Atlantic Ocean.

Our school year begins in September and ends in June. We have a winter break in December. It is
strange to think that you are having winter when we are having summer. Do you get snow in the
winter?

My daily routine is busy, and some of the things I do each day are similar to what you do. On
weekdays, I get up early and eat waffles with fruit. Some of the blueberries that my mom buys come
from Chile!

At 7:15 A.M., my mom and I leave our apartment and walk to the subway station. A subway is an
underground train. The subway ride takes 15 minutes, and there are always people to watch on it.
After the subway, we walk one block to my school, and then Mom continues walking to work.

At 8:15 A.M., my school day starts with math; we are studying multiplication. In language arts, we are
writing speeches. Like you, I also like to write stories, and my favorite topic to write about is horses.
What is the name of your horse?

We also take physical education, music, art, science, and social studies. I enjoy running in physical
education. On weekends, I practice running with my mom. Our class will not begin learning Spanish
until next school year. Can you teach me a few words in your next letter?

I attend an after-school program until my mom is finished with work. We do arts and crafts and play
games there. On Tuesdays and Thursdays, I take violin lessons after school. After Mom picks me up,
we ride the subway and arrive home around 6:00 P.M.

We usually eat dinner around 7:00 P.M. After dinner, I do my homework and practice violin. Then I get
ready for bed at 9:00 P.M. I enjoy listening to classical music as I fall asleep.

I look forward to reading more about Chile!

Your friend,

Mia Johnson

Page 52 of 152 Georgia Milestones Grade 4 EOG Study/Resource Guide for Students and Parents
Copyright © 2020 by Georgia Department of Education. All rights reserved.
English Language Arts (ELA)

Item 16
Selected-Response
Read the sentence from Mia’s letter.

My daily routine is busy, and some of the things I do each day are similar to what you do.

What does routine MOST LIKELY mean?

A. pastime
B. regular schedule
C. school lesson
D. talent

Item 17
Selected-Response
Which idea about Franco and Mia is suggested by the first paragraph of BOTH letters?

A. They are meeting through their letters.


B. They really like the places they live.
C. They plan to see each other in person.
D. They will be visiting each other’s country.

Item 18
Selected-Response
Which statement BEST describes how BOTH Franco and Mia organize the ideas in their letters?

A. They compare how their daily lives are the same as and different from one another.
B. They sequence the events of their day by telling what activities they do from morning to night.
C. They explain what caused them to write to each other and the effects of their new friendship.
D. They address the problem of speaking different languages and come up with a solution.

Georgia Milestones Grade 4 EOG Study/Resource Guide for Students and Parents Page 53 of 152
Copyright © 2020 by Georgia Department of Education. All rights reserved.
English Language Arts (ELA)

Item 19
Extended Constructed-Response
Imagine that Mia gets to visit Franco in Chile or that Franco gets to visit Mia in New York City. Write
an original story about their visit together. Use ideas from the letters and dialogue in your story.

Narrative Writer’s Checklist


Be sure to:
•• Develop a real or imagined experience.
•• Establish a situation and introduce a narrator and/or characters.
•• Organize events in order.
◦◦ Use transitional words and phrases to sequence the events.
•• Use dialogue and/or description to:
◦◦ develop events.
◦◦ show how characters respond to situations.
•• Use concrete words, phrases, and sensory details to describe the events.
•• Include a conclusion.
•• Use ideas and/or details from the passage(s).
•• Check your work for correct usage, grammar, spelling, capitalization, and punctuation.

Now write your narrative on your answer document. Refer to the Writer’s Checklist as you write and
proofread your narrative.

Page 54 of 152 Georgia Milestones Grade 4 EOG Study/Resource Guide for Students and Parents
Copyright © 2020 by Georgia Department of Education. All rights reserved.
English Language Arts (ELA)

Georgia Milestones Grade 4 EOG Study/Resource Guide for Students and Parents Page 55 of 152
Copyright © 2020 by Georgia Department of Education. All rights reserved.
English Language Arts (ELA)

Sample Items 20–25

Writing Standalone Items


On the Georgia Milestones End-of-Grade assessment, there will be writing standalone items that assess
your understanding of opinion, informational/explanatory, and narrative writing and revision skills. There
will also be writing standalone items that assess your writing planning and research skills.

Item 20
Selected-Response
Read the paragraph from a student’s opinion essay.

Out of all the types of books people can read, I think comic books are the best. The stories
have exciting plots, and I enjoy reading about the adventures of superheroes and villains.
______________, there are interesting illustrations that help bring the stories to life. For these
reasons, I think people should check out comic books from the library whenever they can.

Which linking word or phrase BEST fills in the blank to connect the ideas in the paragraph?

A. Instead
B. For instance
C. In addition
D. On the other hand

Item 21
Selected-Response
Read the paragraph from a student’s report.

Fruits do not come from the same part of the plant as vegetables. Fruits come from any part
of the plant that contains a seed. Some popular fruits are apples, strawberries, and bananas.
If you look carefully, you’ll notice that each of these fruits has seeds. Fruits can usually be
enjoyed without being cooked. Unlike fruits, vegetables do not have seeds. They come from
other parts of a plant that can be eaten, such as the roots, stems, leaves, and flower buds. For
example, lettuce comes from the leaf of a plant, while potatoes come from the root of a plant.
Vegetables can be eaten raw, but many people prefer to eat them cooked.

Which sentence would BEST introduce the ideas in the paragraph?

A. It is important to eat fruits and vegetables as part of a balanced diet.


B. Though both fruits and vegetables come from plants, they differ in many ways.
C. Both fruits and vegetables are plants that can be used in a wide variety of dishes.
D. While fruits tend to taste sweet, vegetables usually have a milder flavor.

Page 56 of 152 Georgia Milestones Grade 4 EOG Study/Resource Guide for Students and Parents
Copyright © 2020 by Georgia Department of Education. All rights reserved.
English Language Arts (ELA)

Item 22
Selected-Response
Read the paragraph from a student’s story.

I awoke early this morning so that I could take the time to be extra careful in packing my
backpack. This was the first time my family was going camping. Yesterday, my sister told me
the campsite was three hours away from home. I wanted to be certain that I did not leave
anything important behind. After I finished packing, I grabbed my sleeping bag and pillow
and headed down to the kitchen for breakfast. I heard my parents and sister moving around
upstairs as I poured myself a bowl of cereal.

Which sentence should be added to help introduce the narrator of the story?

A. I called up to my sister, “Tasha, what do you want to eat for breakfast? I’ll start to make it for you.”
B. I was so eager to go camping that I felt like days had passed before my family finally came downstairs
to join me for breakfast.
C. My mom called down to me, “Mike, since you’re already in the kitchen, could you please press the
start button on the coffee machine?”
D. While my family was finishing their own packing, I went into the garage and began to put some of our
camping gear into the van.

Item 23
Selected-Response
Read the paragraph from a student’s report.

The Mississippi River is the second-longest river in North America. The river has served many
purposes throughout history. Native Americans lived along the river and used it as a source
of water, food, and transportation. Early explorers and traders used boats to travel along the
Mississippi River as well. Over the years, even birds and fish have used the river as a migration
route!

Which source should the student use to gather more information for this part of the report?

A. a book of stories that includes legends about different animals that lived in the Mississippi River many
years ago
B. a textbook with a map that shows the ten different states the Mississippi River flows through
C. an interview with a scientist about how water levels can affect the flow of the Mississippi River
D. an article explaining how the Mississippi River has been used to ship supplies from one place to
another

Georgia Milestones Grade 4 EOG Study/Resource Guide for Students and Parents Page 57 of 152
Copyright © 2020 by Georgia Department of Education. All rights reserved.
English Language Arts (ELA)

Item 24
Drop-Down Technology-Enhanced

ÂÂ
Use a mouse, touchpad, or touchscreen to click the arrow beside each of the two blank boxes. When
you click the arrow, a drop-down menu will appear, showing you all the possible options for that blank.
Each drop-down menu with its options is shown below.

Page 58 of 152 Georgia Milestones Grade 4 EOG Study/Resource Guide for Students and Parents
Copyright © 2020 by Georgia Department of Education. All rights reserved.
English Language Arts (ELA)

Item 25
Drag-and-Drop Technology-Enhanced

ÂÂ
Use a mouse, touchpad, or touchscreen to move the descriptions below the table into the boxes in
the table.

Georgia Milestones Grade 4 EOG Study/Resource Guide for Students and Parents Page 59 of 152
Copyright © 2020 by Georgia Department of Education. All rights reserved.
English Language Arts (ELA)

Unit 4: Language

CONTENT DESCRIPTION
The language portion of the English Language Arts test focuses on the conventions of Standard English,
including grammar and usage and the proper use of capitalization, punctuation, and spelling.

Conventions of Standard English


•• Use correct grammar and usage when writing.
•• Use correct capitalization, punctuation, and spelling.

Knowledge of Language
•• Express yourself clearly and in an interesting way.
•• Choose your words carefully so readers understand what you are writing.

Vocabulary Acquisition and Use


•• Vary the words you use in your writing.
•• Use different strategies (e.g., context, affixes, roots) to help you determine the meaning of unknown or
multiple-meaning words.
•• Show an understanding of figurative language (i.e., similes, metaphors) and word relationships (i.e.,
synonyms, antonyms).
•• Recognize and explain the meaning of common idioms, adages, and proverbs.
•• Use reference materials to determine the precise meanings of words or phrases.

KEY TERMS
Grammar: The system of rules for a language. (L1)
Usage: Using the correct word when there is a choice (e.g., to, too, and two). (L1)
Pronoun: A part of speech that is used instead of a noun when the meaning of the noun is already
understood. I, we, he, she, they, and it are all pronouns. (L1a)
Relative pronouns: Words used to refer to a noun that was already mentioned but is being referred to
again. Examples of relative pronouns are who, which, whose, whom, and that. (L1a)
Adverb: A part of speech that describes a verb, an adjective, or another adverb. Adverbs usually end in -ly.
Quietly, thoroughly, frantically, and lovingly are all adverbs. (L1a)
Relative adverb: A relative adverb is used to give more details in a sentence. For example, where, when,
and why. (L1a)
Progressive tense: A tense used to describe an action that is ongoing and has not stopped. For example,
I am walking, I was walking, and I will be walking are all variations of the progressive tense. (L1b)
Helping verb: A verb that helps a main verb by adding to the meaning of the main verb (e.g., am, are, is,
were, could). For example, in the sentence Dad may plant roses in the garden, the helping verb is may.
(L1c)

Page 60 of 152 Georgia Milestones Grade 4 EOG Study/Resource Guide for Students and Parents
Copyright © 2020 by Georgia Department of Education. All rights reserved.
English Language Arts (ELA)

Linking verb: Connects the subject to a word that describes the subject. For example, in the sentence The
sky is blue, the word is links the subject sky to the word blue, which describes sky. Linking verbs do not
show any action. The most common linking verbs are forms of the verb be: am, is, was, were, are. (L1c)
Condition: Create conditional sentences, which are sentences that describe situations that might happen
but haven’t happened yet. Linking verbs and helping verbs can be added to verbs to show conditions; for
example, If the weather is wet, George will need an umbrella. The helping verb will works with need to
explain the type of condition that would require George to need an umbrella. (L1c)
Adjective: A part of speech that describes a noun. Beautiful, tall, blue, and interesting are all adjectives.
(L1d)
Order of adjectives: This refers to the order in which adjectives are correctly listed according to their type;
for example, the big red ball. (L1d)
Prepositional phrases: Phrases that are used to show direction, location, or time. Examples of
prepositional phrases are on the box, in the box, around the box, by the box, and through the box. (L1e)
Sentence fragment: A sentence that is incomplete. A short walk would be a sentence fragment. The
complete sentence would be I went on a short walk. (L1f)
Run-on sentence: A run-on happens when more than one sentence is joined without the correct word
and/or punctuation mark. (L1f)
Capitalization: To correctly make the first letter of a word uppercase. (L2, L2a)
Punctuation: Writing marks that help to separate and clarify ideas. Examples of punctuation are periods,
colons, exclamation marks, and question marks. Commas and quotation marks are used for dialogue to
show the exact words being said. Commas are also used before coordinating conjunctions in a compound
sentence. Punctuation can also be used for effect. For example, an exclamation mark can be used to show
excitement. (L2, L2b, L2c, L3b)
Quotation: Repeating or writing out exactly what a source said, word for word. Quotes are always put
inside quotation marks. (L2b)
Coordinating conjunction: A word that is used to combine two simple sentences to make a compound
sentence. For example, and, or, but. (L2c)
Compound sentence: A compound sentence contains two independent clauses joined by a conjunction.
An independent clause is a part of a sentence that can stand alone because it expresses a complete
thought and has a subject and a verb. An example of a compound sentence is The child rode his bicycle to
school, so he made it to his first class on time. The sentence contains two independent clauses joined by
the conjunction so. (L2c)
Conventions: Rules for how to spell words, write sentences, and use punctuation so that everyone who
reads or speaks that language will understand the intended meaning. For example, capitalizing the first
word of a sentence is a convention of the English language. (L3)
Formal style: A formal style is used in formal writing, such as an essay, research paper, or formal letter.
When writing in a formal style, the writer chooses language that matches the audience and purpose and
avoids informal language. (L3c)
Context clues: The words, facts, or ideas in a text that help you understand the meaning of an unknown
word. (L4, L4a)

Georgia Milestones Grade 4 EOG Study/Resource Guide for Students and Parents Page 61 of 152
Copyright © 2020 by Georgia Department of Education. All rights reserved.
English Language Arts (ELA)

Context: Words and phrases that surround an unknown word or phrase and help to explain its meaning.
Sometimes a word cannot be understood without the context of the words and phrases around it. For
example, the word sink is a multiple-meaning word because it could mean several things. The meaning
is clear when the full sentence is included: She will throw the basketball up high from midcourt and sink it
through the hoop for three points. (L4, L4a)
Root word: The base word. Knowing the meaning of the root word can help a reader determine the
meaning of other forms of the word. For example, if you know that the root word school is a place that
provides knowledge, you may be able to guess that a scholar is someone who is seeking knowledge. (L4b)
Affix: Letters added to a root word that change its meaning. For example, when the prefix dis- is added to
the word interest, the word disinterest means the opposite of the root word interest. (L4b)
Dictionary: A reference book that provides the precise, or exact, meanings of words and phrases. (L4c)
Glossary: An alphabetical list of words and phrases and their meanings. A glossary is often found at the
end of a text. (L4c)
Figurative language: To understand figurative language, you cannot simply define the words in the phrase.
You will need to distinguish between literal and figurative meanings of words and phrases. (Literal refers to
the “actual meaning of a word or phrase.”) For example, if someone tells you to “open the door,” you can
open a real door. If someone tells you to “open the door to your heart,” you are not expected to find a door
in your chest. Instead, you are to open up your feelings and emotions. (L5)

•• Simile: A comparison using like or as; for example, “She is as pretty as a picture.” (L5a)
•• Metaphor: A direct comparison that states one thing is another. It isn’t meant to be literal, but
descriptive. For example, if someone describes recess by saying “It was a zoo,” he or she is using
a metaphor. Recess was chaotic, with many different people running around; it was not literally a
zoo. (L5a)
•• Adage: A saying that is repeated and is generally accepted as truth over time. An example is, “A
penny saved is a penny earned.”(L5b)
•• Proverb: A short saying that gives a piece of advice, such as “Don’t rock the boat.” (L5b)
•• Idioms: Quirky sayings and expressions specific to a language. For example, “Solving that puzzle
was a piece of cake” means that the puzzle was easy, not that it was something to be eaten. If
a saying seems unfamiliar or is not understood, it may be an idiom that needs to be researched.
(L5b)

Synonyms: Words that have the same meaning. Small and little are synonyms. (L5c)
Antonyms: Words that have opposite meanings. Small and large are antonyms. (L5c)

Important Tips
"" To study for this part of the EOG assessment, concentrate on the kinds of errors you typically make in
your own writing. Then review grammar rules for those specific kinds of errors. Use books or free online
resources to find practice items that you can try. You can work with a partner and question each other
on grammar rules or try editing sentences together. Focus your review time on strengthening the areas
or skills that need it the most.
"" When you are faced with an unknown word, go back to the passage. Start reading two sentences
before the word appears, and continue reading for two sentences afterward. If that doesn’t give you
enough clues, look elsewhere in the passage.

Page 62 of 152 Georgia Milestones Grade 4 EOG Study/Resource Guide for Students and Parents
Copyright © 2020 by Georgia Department of Education. All rights reserved.
English Language Arts (ELA)

Sample Items 26–35

Item 26
Selected-Response
Which form of the verb correctly completes the sentence?

Roger when suddenly he heard a knock on the door.

A. is reading
B. was reading
C. will be reading
D. has been reading

Item 27
Selected-Response
In which sentence are the adjectives ordered correctly?

A. Ted’s mother drove a tiny old car.


B. Melissa lived in a blue large house.
C. Henry listened to a French tall man singing.
D. There was a copper strange handle on Cliff’s door.

Item 28
Selected-Response
Which sentence shows correct use of a prepositional phrase?

A. I dropped the ball, and it rolled in the creek.


B. The rain came down of the sky like a waterfall.
C. Paula pulled the rock out of the water and dried it.
D. The young parrot left its cage and flew out from the house.

Georgia Milestones Grade 4 EOG Study/Resource Guide for Students and Parents Page 63 of 152
Copyright © 2020 by Georgia Department of Education. All rights reserved.
English Language Arts (ELA)

Item 29
Selected-Response
In which sentence is the underlined word used correctly?

A. All the students brought their books to school.


B. The extra work helped improve my grades to.
C. Everyone went to Daniela’s party accept for Roland.
D. There is nothing better then cold ice cream on a hot day.

Item 30
Selected-Response
Which sentence uses correct punctuation?
A. Jenna asked her father “Who is your favorite singer?”
B. Jenna asked her father? “Who is your favorite singer?”
C. Jenna asked her father. “Who is your favorite singer?”
D. Jenna asked her father, “Who is your favorite singer?”

Item 31
Selected-Response
Which sentences use would, could, and should correctly?

A. We would try to find the person who owns this dog. We should put up signs, and hopefully the dog’s
owner could see them.
B. We should try to find the person who owns this dog. We could put up signs, and hopefully the dog’s
owner would see them.
C. We could try to find the person who owns this dog. We would put up signs, and hopefully the dog’s
owner should see them.
D. We would try to find the person who owns this dog. We could put up signs, and hopefully the dog’s
owner should see them.

Page 64 of 152 Georgia Milestones Grade 4 EOG Study/Resource Guide for Students and Parents
Copyright © 2020 by Georgia Department of Education. All rights reserved.
English Language Arts (ELA)

Item 32
Selected-Response
Read the story that a student is writing.
1
Yesterday, two zookeepers visited my school to give a presentation. 2They told us facts about
many kinds of animals. 3Then one of the zookeepers brought out a surprise from behind a
curtain. 4It was a real-life version of our school mascot—an eagle. 5I couldn’t believe it. 6By the
end of the presentation, I had a new interest in eagles.

Which sentence should end with an exclamation point to help show the narrator’s excitement?

A. sentence 3
B. sentence 4
C. sentence 5
D. sentence 6

Item 33
Selected-Response
Read the paragraph.

In soccer, the goalie must defend the goal. The entyre team depends on the goalie to do this.
It is a problem if the goalie stops paying attention to the game.

Which word is misspelled?

A. defend
B. entyre
C. problem
D. attention

Georgia Milestones Grade 4 EOG Study/Resource Guide for Students and Parents Page 65 of 152
Copyright © 2020 by Georgia Department of Education. All rights reserved.
English Language Arts (ELA)

Item 34
Drop-Down Technology-Enhanced

ÂÂ
Use a mouse, touchpad, or touchscreen to click the arrow beside each of the two blank boxes. When
you click the arrow, a drop-down menu will appear, showing you all the possible options for that blank.
Each drop-down menu with its options is shown below.

Page 66 of 152 Georgia Milestones Grade 4 EOG Study/Resource Guide for Students and Parents
Copyright © 2020 by Georgia Department of Education. All rights reserved.
English Language Arts (ELA)

Item 35
Drop-Down Technology-Enhanced

ÂÂ
Use a mouse, touchpad, or touchscreen to click the arrow beside each of the three blank boxes.
When you click the arrow, a drop-down menu will appear, showing you all the possible options for that
blank. Each drop-down menu with its options is shown below.

Georgia Milestones Grade 4 EOG Study/Resource Guide for Students and Parents Page 67 of 152
Copyright © 2020 by Georgia Department of Education. All rights reserved.
English Language Arts (ELA)

ENGLISH LANGUAGE ARTS (ELA) ADDITIONAL SAMPLE ITEM KEYS

Standard/
DOK Correct
Item Element/ Explanation
Level Answer
Genre
The correct answer is choice (B) She knows Greta is upset
and is trying to comfort her. This answer is supported by
Greta’s mother’s earlier comment, “Yes, and it will all be
fine.” Choice (A) is incorrect because there is no evidence
ELAGSE4RL3 that Greta’s mother wants to teach Greta to play the piano.
1 2 B Choice (C) is incorrect because Greta’s mother mentions
Literary she knows how to play the piano, but there is no evidence
that she would like to explain how she learned to play.
Choice (D) is incorrect because although Greta knows her
neighbors are moving, the story doesn’t provide enough
evidence to show that their moving is Greta’s main concern.
The correct answer is choice (C) Greta’s mother stopped
playing the piano bit by bit. Perhaps unintentionally, Greta’s
mother slowly moved away from playing piano. Choice (A) is
incorrect because there is no evidence that Greta’s mother
ELAGSE4RL4 stopped enjoying music; she just got out of the habit of
2 2 C
Literary playing it. Choice (B) is incorrect because the passage
contains no evidence that music made Greta’s mother sad.
Choice (D) is incorrect because “drifting away” refers to
abandoning piano playing altogether, not discontinuing to
listen in the middle of a song.
The correct answer is choice (D) She enjoys hearing her
mother playing the piano. The context of the passage
supports the idea that Greta enjoys listening to her mother
play and that knowing that her mother can play is like “a
ELAGSE4RL1 secret,” causing “a bubble of warmth [to rise] inside her.”
3 2 D
Literary Choice (A) is incorrect because there is no suggestion it
is important to Greta that others hear her mother play.
Choice (B) is incorrect because she is not trying to nudge
her mother into doing a difficult task. Choice (C) is incorrect
because her mother seems to be an adept pianist.
The correct answer is choice (A) to show how Greta’s
feelings are changing. There are two sections of dialogue,
with the first showing Greta’s dismay at the piano’s arrival
ELAGSE4RL5 and the second showing Greta’s pleasure at her mother’s
4 2 A piano playing. Choice (B) is incorrect because Greta’s
Literary mother is not upset. Choice (C) is incorrect because Greta’s
emotions are directed at the piano, not the neighbors.
Choice (D) is incorrect because Greta’s mother reassures
her daughter but does not express worry over her.

Page 68 of 152 Georgia Milestones Grade 4 EOG Study/Resource Guide for Students and Parents
Copyright © 2020 by Georgia Department of Education. All rights reserved.
English Language Arts (ELA)

Standard/
DOK Correct
Item Element/ Explanation
Level Answer
Genre
The correct answer is choice (A) The story is told by a
narrator who provides Greta’s point of view. The story is told
ELAGSE4RL6 from a third-person limited point of view, limited to Greta’s
5 2 A
Literary thoughts and emotions. Choices (B), (C), and (D) are
incorrect because the story is not told from the mother’s,
Richard’s, or an omniscient point of view.
The correct answers are choice (C) talented and choice (B)
The sound was pouring out of her mother’s fingers, but she
was looking straight ahead with her head cocked slightly to
the right.
Greta hears beautiful music and cannot identify the source
right away; she is pleasantly surprised to find that it is her
mother playing the piano. The correct answer choice for
ELAGSE4RL3 Part B of the item shows text that supports this.
6 3 C/B
Literary In Part A, choice (A) is incorrect because, while it is
appealing, Greta’s mother’s talent with the piano is
emphasized more in the passage. Choice (B) is incorrect
as there is no indication that Greta’s mother is humorous.
Choice (D) is incorrect because there is no scenario that
requires Greta’s mother to be forgiving within the passage.
The incorrect options in Part B support incorrect answers in
Part A.
ELAGSE4RL2
7 3 N/A See scoring rubric and exemplar response on page 75.
Literary
See scoring rubric beginning on page 85 and sample
8 ELAGSE4W3 4 N/A
responses on page 76.
The correct answer is choice (D) New Yorkers didn’t want
to be left out. The key word in the question is finally. It was
Bartholdi’s “good idea” that inspired donations to complete
ELAGSE4RI1 the base. Choice (A) is incorrect because it refers to the
9 Informational/ 2 D time before fundraising began in the United States.
Explanatory Choice (B) is incorrect because Americans’ initial
excitement inspired donations for the statue, not the base.
Choice (C) is incorrect because this also refers to the
statue rather than the base.

Georgia Milestones Grade 4 EOG Study/Resource Guide for Students and Parents Page 69 of 152
Copyright © 2020 by Georgia Department of Education. All rights reserved.
English Language Arts (ELA)

Standard/
DOK Correct
Item Element/ Explanation
Level Answer
Genre
The correct answer is choice (D) told a lot of people.
Bartholdi wanted to make sure New Yorkers heard what
he was saying. Choice (A) is incorrect because the author
ELAGSE4RI4 never states that Bartholdi was lying. Choice (B) is incorrect
10 Informational/ 2 D because “spread the word” means the opposite of keeping
Explanatory a secret; it means telling a lot of people. Choice (C) is
incorrect because although a sign could help spread the
word, there is no evidence that Bartholdi limited his efforts
to a sign.
The correct answer is choice (A) People often do not know
the real story behind events. The author writes, “Many
people know that . . . , [b]ut the story is not that simple.”
Choice (B) is incorrect because the author provides
ELAGSE4RI8 information about how the statue came to be built.
11 Informational/ 2 A Choice (C) is incorrect because although some of the
Explanatory money came from France, a significant portion came from
the United States. Choice (D) is incorrect because the
author does not imply that Americans don’t care about their
history; on the contrary, they donated money to make the
statue.
The correct answer is choice (C) Weekend homework might
help students get into college. The author mentions that
students who aspire to go to college would “do whatever
ELAGSE4RI8 is necessary” to get into college, and the author’s overall
purpose is to encourage weekend homework. Choice (A)
12 Informational/ 2 C
is incorrect because the author mentions nothing about
Explanatory college students’ jobs. Choices (B) and (D) are incorrect
because although they could be used to support the
author’s argument, they are less directly related to the
author’s final argument than choice (C) is.
The correct answer is choice (D) Tired, unhappy students
don’t perform well. This suggests that if students did
no weekend homework, they would be neither tired nor
unhappy, and they would perform better than if they had
ELAGSE4RI1 done weekend homework. Choices (A) and (B) are incorrect
13 Informational/ 2 D because even if homework has no effect, we cannot
Explanatory logically conclude that students would do better in school
if they did no weekend homework; they might perform in
exactly the same way. Choice (C) is incorrect because it
bears no relevance to the question. The author makes no
connection between family time and school performance.

Page 70 of 152 Georgia Milestones Grade 4 EOG Study/Resource Guide for Students and Parents
Copyright © 2020 by Georgia Department of Education. All rights reserved.
English Language Arts (ELA)

Standard/
DOK Correct
Item Element/ Explanation
Level Answer
Genre
The correct answer is choice (D) It is important to have
time to play on the weekends. This point is mentioned in
ELAGSE4RI1 both articles. Choice (A) is incorrect because the articles
don’t say homework is important for young kids. Choice (B)
14 Informational/ 3 D
is incorrect because only one of the articles talks about
Explanatory students’ futures. Choice (C) is incorrect because no
homework on the weekend is supported by only one of the
articles.
ELAGSE4W1,
See scoring rubric beginning on page 89 and sample
15 ELAGSE4L1, 4 N/A
responses on page 77.
ELAGSE4L2
The correct answer is choice (B) regular schedule. The
sentence-level context “some things I do each day”
supports regular schedule as the meaning of the word
routine. Choice (A) is incorrect because the activities Mia
16 ELAGSE4L4a 2 B
enjoys doing in her free time are not activities she does
daily. Choice (C) is incorrect because Mia attends school
only on weekdays, not daily. Choice (D) is incorrect because
talent is not supported by the context in this sentence.
The correct answer is choice (A) They are meeting
through their letters. Franco introduces himself in the first
paragraph of his letter, and Mia says, “It is interesting to
meet someone who lives there,” in response to his letter
17 ELAGSE4RI9 3 A about living in Chile. Choice (B) is incorrect because the
details in Mia’s first paragraph do not reference where she
lives. Choices (C) and (D) are incorrect because neither first
paragraph indicates they plan to visit each other or each
other’s country.
The correct answer is choice (B) They sequence the events
of their day by telling what activities they do from morning
to night. Both Franco and Mia walk through a typical day
in chronological order. Choice (A) is incorrect because the
18 ELAGSE4RI5 3 B structure of each letter is not compare and contrast.
Choice (C) is incorrect because the structure of each letter
is not cause and effect. Choice (D) is incorrect because
neither letter indicates that the fact that they speak
different languages at home is a problem.
See scoring rubric beginning on page 85 and sample
19 ELAGSE4W3 4 N/A
responses beginning on page 78.

Georgia Milestones Grade 4 EOG Study/Resource Guide for Students and Parents Page 71 of 152
Copyright © 2020 by Georgia Department of Education. All rights reserved.
English Language Arts (ELA)

Standard/
DOK Correct
Item Element/ Explanation
Level Answer
Genre
The correct answer is choice (C) In addition. The linking
phrase connects an additional reason why the author
thinks comic books are the best type of book. Choice (A) is
incorrect because it indicates the sentence is a contrasting
20 ELAGSE4W1c 2 C idea, which is not accurate. Choice (B) is incorrect because
it indicates the sentence is an example of the previous
sentence, but it is an additional idea. Choice (D) is
incorrect because it indicates the sentence is a contrasting
idea, which is not accurate.
The correct answer is choice (B) Though both fruits and
vegetables come from plants, they differ in many ways. The
paragraph provides details about how fruits and vegetables
come from different parts of a plant and can be prepared
in different ways. Choice (A) is incorrect because, although
the paragraph mentions eating fruits and vegetables, it
21 ELAGSE4W2a 2 B
does not focus on their dietary benefits. Choice (C) is
incorrect because examples of recipes that include fruits
and vegetables are not discussed in the paragraph.
Choice (D) is incorrect because, rather than introducing the
topic, the sentence is an additional detail comparing fruits
and vegetables.
The correct answer is choice (C) My mom called down to
me, “Mike, since you’re already in the kitchen, could you
please press the start button on the coffee machine?” This
is the only option that tells the reader who the narrator is.
Choice (A) is incorrect because the sentence introduces
22 ELAGSE4W3a 3 C
the narrator’s sister. Choice (B) is incorrect because it
describes how the narrator feels but does not identify
who the narrator is. Choice (D) is incorrect because the
sentence is an additional story detail and does not identify
the narrator.
The correct answer is choice (D) an article explaining
how the Mississippi River has been used to ship supplies
from one place to another. This is the only option that is
an additional example of a purpose the river has served.
Choice (A) is incorrect because the source is fiction and
23 ELAGSE4W8 3 D
would not support a research paper. Choice (B) is incorrect
because the map does not support that the river has
different purposes. Choice (C) is incorrect because the
interview describes a process rather than explaining an
additional purpose of the river.
24 ELAGSE4W3d 2 N/A See scoring rubric and exemplar response on page 80.
25 ELAGSE4W7 2 N/A See scoring rubric and exemplar response on page 81.

Page 72 of 152 Georgia Milestones Grade 4 EOG Study/Resource Guide for Students and Parents
Copyright © 2020 by Georgia Department of Education. All rights reserved.
English Language Arts (ELA)

Standard/
DOK Correct
Item Element/ Explanation
Level Answer
Genre
The correct answer is choice (B) was reading. Reading was
an activity that was taking place when Roger heard the
knock. Choice (A) is incorrect because it is the wrong tense
26 ELAGSE4L1b 2 B of the verb. Choice (C) is incorrect because the action of
the entire sentence occurred in the past, not the future.
Choice (D) is incorrect because it is the wrong tense of the
verb.
The correct answer is choice (A) Ted’s mother drove a tiny
old car. Size-related adjectives come before age-related
adjectives. Choice (B) is incorrect because size comes
27 ELAGSE4L1d 2 A
before color. Choice (C) is incorrect because national origin
comes after size. Choice (D) is incorrect because opinion
comes before material.
The correct answer is choice (C) Paula pulled the rock out
of the water and dried it. The phrase “out of the water”
is a correctly used prepositional phrase. Choice (A) is
incorrect because the correct preposition of motion in
28 ELAGSE4L1e 2 C
this use is into, not in. Choice (B) is incorrect because the
prepositional phrase should read “from the sky.” Choice (D)
is incorrect because the prepositional phrase should be “of
the house.”
The correct answer is choice (A) All the students brought
their books to school. The writer does not make the
common mistake of using there. Choice (B) is incorrect
because the correct word is too, not to. Choice (C) is
29 ELAGSE4L1g 2 A
incorrect because the correct word is except; accept does
not make sense. Choice (D) is incorrect because the correct
word is than—a word of comparison rather than a word of
sequence.
The correct answer is choice (D) Jenna asked her father,
“Who is your favorite singer?” This sentence provides a
comma, which is the appropriate punctuation before the
30 ELAGSE4L2b 2 D
quotation. Choices (A), (B), and (C) are incorrect because a
lack of punctuation, a question mark, and a period are all
incorrect punctuation choices before the quotation.
The correct answer is choice (B) We should try to find the
person who owns this dog. We could put up signs, and
31 ELAGSE4L1c 2 B hopefully the dog’s owner would see them. Choices (A), (C),
and (D) all use would, could, and should in ways that do not
sensibly combine the two sentences.

Georgia Milestones Grade 4 EOG Study/Resource Guide for Students and Parents Page 73 of 152
Copyright © 2020 by Georgia Department of Education. All rights reserved.
English Language Arts (ELA)

Standard/
DOK Correct
Item Element/ Explanation
Level Answer
Genre
The correct answer is choice (C) sentence 5. This option
is correct because the exclamation mark supports the
student’s enthusiasm for what was seen during the
32 ELAGSE4L3b 2 C presentation. Choices (A) and (D) are incorrect because
they are matters of fact. Choice (B) is incorrect because
it is the sentence that is leading into the narrator’s
excitement, but it does not require an exclamation mark.
The correct answer is choice (B) entyre. This word should
be correctly spelled entire. Choices (A), (C), and (D) are
33 ELAGSE4L2d 1 B
incorrect because defend, problem, and attention are all
spelled correctly.
34 ELAGSE4L3a 2 N/A See scoring rubric and exemplar response on page 82.
35 ELAGSE4L1e 2 N/A See scoring rubric and exemplar response on page 83.

Page 74 of 152 Georgia Milestones Grade 4 EOG Study/Resource Guide for Students and Parents
Copyright © 2020 by Georgia Department of Education. All rights reserved.
English Language Arts (ELA)

ENGLISH LANGUAGE ARTS (ELA) EXAMPLE SCORING RUBRICS AND


EXEMPLAR RESPONSES
Item 7
Scoring Rubric

Points Description
2 The student correctly fills in all three rows.
1 The student correctly fills in two rows.
0 The student does not correctly fill in at least two rows.

Exemplar Response
The correct response is shown below.

The correct response for Event 2 is “Greta is upset about the piano, so she runs to her room.” The correct
response for Event 3 is “Greta awakes from a nap and hears beautiful piano music.” The correct response
for Event 4 is “Greta is happy to learn that her mom can play the piano.” These are the correct responses
because each sentence represents a key event that occurred in the story. In addition, each sentence is
correctly placed in the order the events occurred in the story.

Georgia Milestones Grade 4 EOG Study/Resource Guide for Students and Parents Page 75 of 152
Copyright © 2020 by Georgia Department of Education. All rights reserved.
English Language Arts (ELA)

Item 8
To view the four-point holistic rubric for a text-based narrative response, see pages 85 and 86.
Exemplar Response

Points
Sample Response
Awarded
Greta’s mother said, “Now tell me why you were so upset when you saw the piano.”
“Well, for one thing, you know me. I don’t like surprises.”
“That’s for sure! I did know that,” her mother said.
“Well, then Richard was pounding on the piano really loudly. And it replaced my toy chest.
My toy chest has always been in that spot in the living room.” Greta felt a little foolish for a
4 moment. She giggled softly.
“Well,” said her mother, raising her eyebrows, “we could always bring the toy chest back in
here and get rid of the piano. Then you can play with your stuffed animals all day.”
They both laughed.
“Now let me show you a simple little melody,” Greta’s mother said, turning to the piano
keys.
Greta’s mother said, “Now tell me why you were so upset when you saw the piano.”
“Because I don’t like change. And this thing was a big change.”
3 “Okay,” said her mother. “I understand. I know the piano is a big change. I wonder if we can
turn it into something good. Do you want to give it a try?”
Her mother then gave Greta her first piano lesson.
Greta’s mother wondered why Greta was so upset about the piano. She knew Greta didn’t
2 like change but thought the piano was a good thing. She decided to play the piano to make
Greta feel better.
Greta’s mother didn’t know why Greta was so upset. She saw Greta run to her room after
1
Richard played the piano.
0 Greta’s mother loves the new piano.

Page 76 of 152 Georgia Milestones Grade 4 EOG Study/Resource Guide for Students and Parents
Copyright © 2020 by Georgia Department of Education. All rights reserved.
English Language Arts (ELA)

Item 15
The following are examples of seven-point responses. See the seven-point, two-trait rubric for a
text‑based opinion response on pages 89 and 90 to see why these examples would earn the
maximum number of points.
Many kids complain if they have homework on the weekend. Some say it is even harmful. However, I
agree with the writer of “Homework on the Weekend” that weekend homework is an important part of our
education.
First of all, homework helps us learn. As the writer points out, learning “doesn’t happen just during the
school week.” There is not enough time during the school day to learn everything we need to know. That’s
why teachers give us homework on the weekend. We can practice what we are learning so we understand it
better.
Secondly, homework also helps get us ready for college and the real world. Just like adults, we have to
learn to work even when we don’t really feel like it. We will learn that if we do our work quickly, we have
more time for fun.
Finally, there is no reason why we can’t do our homework and have fun too. I agree with the writer of
“Weekends Are for Fun” that kids need time for sports and hobbies, and need to spend time with family.
But as the writer says in “Homework on the Weekend,” an hour or so of homework is not too much. There
is still enough time for fun activities and relaxing on the weekends.
In conclusion, I think it is fine for students to have homework on the weekend. It is another way to help
us learn and will help prepare us for the real world. If we plan our time well and work hard, we can get our
homework done and still have time for fun.
OR
Should kids have homework on the weekend? People have different opinions about this topic. I agree with
the writer of “Weekends Are for Fun” that students should not get homework on the weekend.
One reason kids get homework is because people think it helps us learn. But, according to the passage,
a study showed that homework does not help kids do better in school. Even worse, it causes stress and
takes time away from sports, hobbies, and being with family.
Another reason kids get homework is that people think it prepares us for the real world. But we are kids,
not adults. As the writer says in “Weekends Are for Fun,” kids “should forget about school. They should
just be kids.”
In “Homework on the Weekend,” the writer says that one hour of homework on the weekend is fine. But for
many students, homework takes longer than an hour, especially if the work is more difficult for them. They
may not have enough time to relax. They will feel tired on Monday morning when it is time to get up for
school.
For these reasons, I think that kids should not have homework on the weekend. Homework does not help
us learn and it does not prepare us for the real world. In fact, it can even be harmful because it causes
stress. Instead of spending weekends doing homework, we need time to relax and just be kids.

Georgia Milestones Grade 4 EOG Study/Resource Guide for Students and Parents Page 77 of 152
Copyright © 2020 by Georgia Department of Education. All rights reserved.
English Language Arts (ELA)

Item 19
To view the four-point holistic rubric for a text-based narrative response, see pages 85 and 86.
Exemplar Response

Points
Sample Response
Awarded
Mia looked at the picture Franco had sent her in his last letter. It was a picture of Franco
and his horse, Luna. Then she looked up at all the people getting off the plane. She spotted
Franco in the crowd.
“Hola Franco!” she yelled. “Over here.” Franco looked over, saw them and waved. He then
grabbed his mom’s arm and pulled her over towards Mia and her mom.
“Hello Mia,” said Franco. He was smiling.
“Welcome to New York City,” Mia said smiling back at him.
As they walked out of the airport, it started to snow.
“Wow,” said Franco. “We rarely get snow in Loncotraro.”
“Try to catch a snowflake on your tongue,” said Mia. She stuck her tongue out to show
Franco how to do it. They both had fun trying to catch the snowflakes. Finally, Mia’s mom
told them it was time to take the subway to their apartment.
4 They boarded the train and settled into their seats. Franco looked around at all the other
passengers. There were a lot more people in New York City than in his hometown.
Once they got home, Mia asked Franco what he wanted to do.
“You play the violin, right?”
“Yes,” replied Mia.
“Can you play it for me? I’ve never seen anybody play one in person before.”
Mia was nervous because she was not used to playing for her friends, but she did not want
to disappoint Franco. She grabbed her violin and slowly pulled the bow across the strings as
she played her favorite song.
“That was incredible!” Franco said when Mia finished, “Can you play another song for me!”
Mia raised her bow, but her mom interrupted before she could start playing by calling them
to dinner. As they walked in to the kitchen, Mia felt happy that Franco liked her music and
couldn’t wait to play another song for him when they were finished eating.

Go on to the next page to finish item 19.

Page 78 of 152 Georgia Milestones Grade 4 EOG Study/Resource Guide for Students and Parents
Copyright © 2020 by Georgia Department of Education. All rights reserved.
English Language Arts (ELA)

Item 19
Exemplar Response, continued

Points
Sample Response
Awarded
Franco felt excited waiting for Mia’s plane to land. He was finally going to meet his pen pal!
“Hola!” Franco heard a voice. He saw a girl and her mother walking toward him. She was
waving at Franco.
“Hello Mia,” he said. They all got on a bus to go to Franco’s house.
“I wrote a story about a horse on the plane,” Mia said. “I tried to use some of the Spanish
3 words you taught me.”
“That’s wonderful! You’ll have to read it to me later,” Franco said.
They arrived at Franco’s house and went inside. Franco’s mom made everyone a sandwich
with honey on it.
“Want to go play soccer?” asked Franco. “Yes,” said Mia. Franco asked his siblings to come
play. They all went outside and played until it got dark.
Franco and his mom met Mia at the airport. The three of them rode a city bus to Franco’s
house. Then Franco introdused Mia to his siblings. They all rode horses together and
2
Franco showed Mia the town. Then they decided to right stories. They read them to Francos
parents. It was a fun day.
Franco came too see me in new York and I showd him the city and my hous. We had fun
1
playin together.
0 My favorite sport to play in physical education is soccer.

Georgia Milestones Grade 4 EOG Study/Resource Guide for Students and Parents Page 79 of 152
Copyright © 2020 by Georgia Department of Education. All rights reserved.
English Language Arts (ELA)

Item 24
Scoring Rubric

Points Description

1 The student correctly selects both drop-down menu options.

0 The student does not correctly select both drop-down menu options.

Exemplar Response
The correct response is shown below.

In the first drop-down menu, the correct response is “dashed.” In the second drop-down menu, the correct
response is “wildly.” These are the correct responses because they both reflect the most precise language
to convey the sense of urgency in the student’s story.

Page 80 of 152 Georgia Milestones Grade 4 EOG Study/Resource Guide for Students and Parents
Copyright © 2020 by Georgia Department of Education. All rights reserved.
English Language Arts (ELA)

Item 25
Scoring Rubric

Points Description

2 The student correctly fills in both rows.

1 The student correctly fills in one row.

0 The student does not correctly fill in either row.

Exemplar Response
The correct response is shown below.

The correct response for the first research question is “Electric airplanes are built in a way that produces
almost no pollution.” This is the correct response because the design of the electric airplane reduces
pollution, which will help maintain a healthier environment. The correct response for the second research
question is “Electric airplanes are much quieter than traditional planes.” This is the correct response
because the fact that the electric airplane is quieter than a traditional plane is an example of how electric
airplanes sound compared to traditional planes.

Georgia Milestones Grade 4 EOG Study/Resource Guide for Students and Parents Page 81 of 152
Copyright © 2020 by Georgia Department of Education. All rights reserved.
English Language Arts (ELA)

Item 34
Scoring Rubric

Points Description

1 The student correctly selects both drop-down menu options.

0 The student does not correctly select both drop-down menu options.

Exemplar Response
The correct response is shown below.

In the first drop-down menu, the correct response is “they were greeted by several tiny rabbit footprints
along the trail.” This is the correct response because “greeted” reveals that the hikers are pleased to
find the footprints. In addition, the word “tiny” indicates the size of the footprints and the phrase “along
the trail” gives a precise location for where the footprints were found. In the second drop-down menu,
the correct response is “she sensed a creature watching her from the tall, breezy grass to her right.” This
is the correct response because the phrase “sensed a creature” indicates that Jyl became aware of the
rabbit watching her. In addition, the phrases “tall, breezy” and “to her right” help the reader create a
mental picture of the grass and its specific location.

Page 82 of 152 Georgia Milestones Grade 4 EOG Study/Resource Guide for Students and Parents
Copyright © 2020 by Georgia Department of Education. All rights reserved.
English Language Arts (ELA)

Item 35
Scoring Rubric

Points Description

2 The student correctly selects all three drop-down menu options.

1 The student correctly selects two drop-down menu options.

0 The student does not correctly select at least two drop-down menu options.

Exemplar Response
The correct response is shown below.

In the first drop-down menu, the correct response is “from mine” because it follows the non-comparative
adjective “different.” In the second drop-down menu, the correct response is “into the pool” because the
preposition “into” represents the movement to another location (such as “the pool”). In the third drop-
down menu, the correct response is “for two hours” because the preposition “for” represents a duration of
time rather than a snapshot in time.

Georgia Milestones Grade 4 EOG Study/Resource Guide for Students and Parents Page 83 of 152
Copyright © 2020 by Georgia Department of Education. All rights reserved.
English Language Arts (ELA)

ENGLISH LANGUAGE ARTS (ELA) WRITING RUBRICS


Grade 4 items that are not machine-scored—i.e., constructed-response, extended constructed-response,
and extended writing-response items—are manually scored using either a holistic rubric or a two-trait
rubric.
Four-Point Holistic Rubric
Genre: Narrative
A holistic rubric evaluates one major trait, which is ideas. On the Georgia Milestones EOG assessment, a
holistic rubric is scored from zero to four. Each point value represents the difference in the levels or quality
of the student’s work. To score an item on a holistic rubric, a scorer need only choose the criteria and
associated point value that best represents the student’s work. Increasing point values represent a greater
understanding of the content and, thus, a higher score.

Seven-Point, Two-Trait Rubric


Genre: Opinion or Informational/Explanatory
A two-trait rubric, on the other hand, evaluates two major traits, which are ideas and conventions. On the
Georgia Milestones EOG assessment, a two-trait rubric contains two scales, one for each trait, ranging
from zero to four on one scale (ideas) and zero to three on the other (conventions). A score is given for
each of the two traits, for a total of seven possible points for the item. To score an item on a two-trait
rubric, a scorer must choose the criteria and associated point value for each trait that best represents
the student’s work. The two scores are added together. Increasing point values represent a greater
understanding of the content and, thus, a higher score.
On the following pages are the rubrics that will be used to evaluate writing on the Georgia Milestones
Grade 4 English Language Arts EOG assessment.

Page 84 of 152 Georgia Milestones Grade 4 EOG Study/Resource Guide for Students and Parents
Copyright © 2020 by Georgia Department of Education. All rights reserved.
English Language Arts (ELA)

Four-Point Holistic Rubric


Genre: Narrative

Writing Trait Points Criteria


The student’s response is a well-developed narrative that fully develops a real
or imagined experience based on text as a stimulus.
•• Effectively establishes a situation and introduces a narrator and/or
characters
•• Organizes an event sequence that unfolds naturally
•• Effectively uses narrative techniques, such as dialogue and description, to
develop rich, interesting experiences and events or show the responses of
characters to situations
4
•• Uses a variety of words and phrases consistently to signal the sequence of
events
•• Uses concrete words, phrases, and sensory language consistently and
effectively to convey experiences and events precisely
•• Provides a conclusion that follows from the narrated experiences or events
This trait •• Integrates ideas and details from source material effectively
examines
•• Has very few or no errors in usage and/or conventions that interfere with
the writer’s meaning*
ability to
effectively The student’s response is a complete narrative that develops a real or
develop real imagined experience based on text as a stimulus.
or imagined •• Establishes a situation and introduces one or more characters
experiences •• Organizes events in a clear, logical order
or events
•• Uses narrative techniques, such as dialogue and description, to develop
using experiences and events or show the responses of characters to situations
effective 3
•• Uses words and/or phrases to indicate sequence
techniques,
descriptive •• Uses words, phrases, and details to convey experiences and events
details, and •• Provides an appropriate conclusion
clear event •• Integrates some ideas and/or details from source material
sequences •• Has a few minor errors in usage and/or conventions that interfere with
based on a meaning*
text that has
The student’s response is an incomplete or oversimplified narrative based on
been read. text as a stimulus.
•• Introduces a vague situation and at least one character
•• Organizes events in a sequence but with some gaps or ambiguity
•• Attempts to use a narrative technique, such as dialogue and description,
to develop experiences and events or show the responses of characters to
situations
2
•• Uses occasional signal words to indicate sequence
•• Uses some words or phrases inconsistently to convey experiences and
events
•• Provides a weak or ambiguous conclusion
•• Attempts to integrate ideas or details from source material
•• Has frequent errors in usage and conventions that sometimes interfere
with meaning*

Georgia Milestones Grade 4 EOG Study/Resource Guide for Students and Parents Page 85 of 152
Copyright © 2020 by Georgia Department of Education. All rights reserved.
English Language Arts (ELA)

Four-Point Holistic Rubric


Genre: Narrative
(continued)

Writing Trait Points Criteria


The student’s response provides evidence of an attempt to write a narrative
based on text as a stimulus.
This trait •• Response is a summary of the story
examines
the writer’s •• Provides a weak or minimal introduction of a situation or a character
ability to •• May be too brief to demonstrate a complete sequence of events
effectively •• Shows little or no attempt to use dialogue or description to develop
develop real 1 experiences and events or show the responses of characters to situations
or imagined •• Uses words that are inappropriate, overly simple, or unclear
experiences
•• Provides few, if any, words that convey events
or events
using •• Provides a minimal or no conclusion
effective •• May use few, if any, ideas or details from source material
techniques, •• Has frequent major errors in usage and conventions that interfere with
descriptive meaning*
details, and
clear event The student will receive a condition code for various reasons:
sequences •• Blank
based on a •• Copied
text that has 0
been read. •• Too Limited to Score/Illegible/Incomprehensible
•• Non-English/Foreign Language
•• Off Topic/Off Task/Offensive

*Students are responsible for language conventions learned in their current grade as well as in prior grades. Refer to the language
skills for each grade to determine the grade-level expectations for grammar, syntax, capitalization, punctuation, and spelling.
Also refer to the “Language Progressive Skills, by Grade” chart in the Appendix for those standards that need continued attention
beyond the grade in which they were introduced.

Page 86 of 152 Georgia Milestones Grade 4 EOG Study/Resource Guide for Students and Parents
Copyright © 2020 by Georgia Department of Education. All rights reserved.
English Language Arts (ELA)

Seven-Point, Two-Trait Rubric


Trait 1 for Informational/Explanatory Genre

Writing Trait Points Criteria


The student’s response is a well-developed informative/explanatory text that
examines a topic in depth and conveys ideas and information clearly based on text
as a stimulus.
•• Effectively introduces a topic
Idea •• Effectively develops the topic with multiple facts, definitions, concrete details,
Development, quotations, or other information and examples related to the topic
4
Organization, •• Groups related ideas together to give some organization to the writing
and Coherence •• Effectively uses linking words and phrases to connect ideas within categories of
information
This trait •• Uses precise language and domain-specific vocabulary to explain the topic
examines the •• Provides a strong concluding statement or section related to the information or
writer’s ability explanation presented
to effectively
The student’s response is a complete informative/explanatory text that examines a
establish a
topic and presents information based on text as a stimulus.
controlling
•• Introduces a topic
idea, support
•• Develops the topic with some facts, definitions, and details
the idea with
3 •• Groups some related ideas together to give partial organization to the writing
evidence from
•• Uses some linking words to connect ideas within categories of information, but
the text(s) read, relationships may not always be clear
and elaborate •• Uses some precise language and domain-specific vocabulary to explain the topic
on the idea •• Provides a concluding statement or section
with examples,
illustrations, The student’s response is an incomplete or oversimplified informative/explanatory
text that cursorily examines a topic based on text as a stimulus.
facts, and other
•• Attempts to introduce a topic
details. The
•• Attempts to develop a topic with too few details, but not all of these are supported
writer must
or relevant to the topic
integrate the 2
•• Ineffectively groups some related ideas together
information •• Uses few linking words to connect ideas, but not all ideas are well connected to
from the text(s) the topic
into his/her •• Uses limited language and vocabulary that does not clearly explain the topic
own words •• Provides a weak concluding statement or section
and arrange
The student’s response is a weak attempt to write an informative/explanatory text
the ideas and
that examines a topic based on text as a stimulus.
supporting
•• May not introduce a topic or topic is unclear
evidence (from
•• May not develop a topic
the text[s] 1
•• May be too brief to group any related ideas together
read) in order
•• May not use any linking words to connect ideas
to create
•• Uses vague, ambiguous, or repetitive language
cohesion for an
•• Provides a minimal or no concluding statement or section
informative/
explanatory The student will receive a condition code for various reasons:
essay. •• Blank
•• Copied
0
•• Too Limited to Score/Illegible/Incomprehensible
•• Non-English/Foreign Language
•• Off Topic/Off Task/Offensive

Georgia Milestones Grade 4 EOG Study/Resource Guide for Students and Parents Page 87 of 152
Copyright © 2020 by Georgia Department of Education. All rights reserved.
English Language Arts (ELA)

Seven-Point, Two-Trait Rubric


Trait 2 for Informational/Explanatory Genre

Writing Trait Points Criteria


The student’s response demonstrates full command of language usage
and conventions.
•• Has clear and complete sentence structure, with appropriate range
3
and variety
•• Shows knowledge of language and its conventions when writing
•• Any errors in usage and conventions do not interfere with meaning*
The student’s response demonstrates partial command of language
usage and conventions.
Language Usage
and Conventions •• Has complete sentences, with some variety
2 •• Shows some knowledge of language and its conventions when
This trait examines
writing
the writer’s ability
to demonstrate •• Has minor errors in usage and conventions with no significant effect
control of sentence on meaning*
formation, usage, The student’s response demonstrates weak command of language
and mechanics usage and conventions.
as embodied in
the grade-level •• Has fragments, run-ons, and/or other sentence structure errors
1
expectations of the •• Shows little knowledge of language and its conventions when writing
language standards. •• Has frequent errors in usage and conventions that interfere with
meaning*
The student will receive a condition code for various reasons:
•• Blank
•• Copied
0
•• Too Limited to Score/Illegible/Incomprehensible
•• Non-English/Foreign Language
•• Off Topic/Off Task/Offensive

*Students are responsible for language conventions learned in their current grade as well as in prior grades. Refer to the language
skills for each grade to determine the grade-level expectations for grammar, syntax, capitalization, punctuation, and spelling.
Also refer to the “Language Progressive Skills, by Grade” chart in the Appendix for those standards that need continued attention
beyond the grade in which they were introduced.

Page 88 of 152 Georgia Milestones Grade 4 EOG Study/Resource Guide for Students and Parents
Copyright © 2020 by Georgia Department of Education. All rights reserved.
English Language Arts (ELA)

Seven-Point, Two-Trait Rubric


Trait 1 for Opinion Genre

Writing Trait Points Criteria


The student’s response is a well-developed opinion piece that effectively examines a
topic and supports a point of view, with reasons, clearly based on text as a stimulus.
•• Effectively introduces a topic and clearly states an opinion
•• Creates an effective organizational structure that logically groups ideas and reasons
4
to support the writer’s purpose
•• Provides clear reasons that are supported by facts and details
Idea
Development, •• Uses linking words and phrases effectively to connect opinions and reasons
Organization, •• Provides a strong concluding statement or section related to the opinion presented
and The student’s response is a complete opinion piece that examines a topic and
Coherence supports a point of view based on text.
This trait •• Introduces a topic and states an opinion
examines the •• Provides some organizational structure that groups ideas and reasons to support the
writer’s ability 3
writer’s purpose
to effectively •• Provides reasons that are supported by facts
establish a
•• Uses some linking words to connect opinions and reasons
point of view
•• Provides a concluding statement or section related to the opinion presented
and to support
the opinion The student’s response is an incomplete or oversimplified opinion piece that
with reasons examines a topic and partially supports a point of view based on text.
from the •• Attempts to introduce a topic and state an opinion
text(s) read.
•• Attempts to provide some organization, but structure sometimes impedes the reader
The writer
2 •• Attempts to provide reasons that are sometimes supported by facts
must form an
opinion from •• Uses few linking words to connect opinions and reasons; connections are not always
the text(s) clear
in his/her •• Provides a weak concluding statement or section that may not be related to the
own words opinion
and organize The student’s response is a weak attempt to write an opinion piece that examines a
reasons for the topic and does not support a text-based point of view.
opinion (from
•• May not introduce a topic or state an opinion
text that they
have read) in 1 •• May not have any organizational structure evident
order to create •• May not provide reasons that are supported by facts
cohesion for •• May not use any linking words to connect opinions and reasons
an opinion •• Provides a minimal or no concluding statement or section
essay.
The student will receive a condition code for various reasons:
•• Blank
0 •• Copied
•• Too Limited to Score/Illegible/Incomprehensible
•• Non-English/Foreign Language
•• Off Topic/Off Task/Offensive

Georgia Milestones Grade 4 EOG Study/Resource Guide for Students and Parents Page 89 of 152
Copyright © 2020 by Georgia Department of Education. All rights reserved.
English Language Arts (ELA)

Seven-Point, Two-Trait Rubric


Trait 2 for Opinion Genre

Writing Trait Points Criteria


The student’s response demonstrates full command of language usage
and conventions.
•• Has clear and complete sentence structure, with appropriate range
3
and variety
•• Shows knowledge of language and its conventions when writing
•• Any errors in usage and conventions do not interfere with meaning*
The student’s response demonstrates partial command of language
Language Usage usage and conventions.
and Conventions •• Has complete sentences, with some variety
This trait examines 2 •• Shows some knowledge of language and its conventions when
the writer’s ability writing
to demonstrate •• Has minor errors in usage and conventions with no significant effect
control of sentence on meaning*
formation, usage,
and mechanics The student’s response demonstrates weak command of language
as embodied in usage and conventions.
the grade-level •• Has fragments, run-ons, and/or other sentence structure errors
expectations of 1
•• Shows little knowledge of language and its conventions when writing
the language
standards. •• Has frequent errors in usage and conventions that interfere with
meaning*
The student will receive a condition code for various reasons:
•• Blank
•• Copied
0
•• Too Limited to Score/Illegible/Incomprehensible
•• Non-English/Foreign Language
•• Off Topic/Off Task/Offensive

*Students are responsible for language conventions learned in their current grade as well as in prior grades. Refer to the language
skills for each grade to determine the grade-level expectations for grammar, syntax, capitalization, punctuation, and spelling.
Also refer to the “Language Progressive Skills, by Grade” chart in the Appendix for those standards that need continued attention
beyond the grade in which they were introduced.

Page 90 of 152 Georgia Milestones Grade 4 EOG Study/Resource Guide for Students and Parents
Copyright © 2020 by Georgia Department of Education. All rights reserved.
English Language Arts (ELA)

ACTIVITY
The following activity develops skills in Unit 1: Reading Literary Text.
Standards: ELAGSE4RL1, ELAGSE4RL2, ELAGSE4RL3, ELAGSE4RL9

Prepare for the activity.


1. Ask a family member to help you locate and print out 10 fables, including “The Lion and the Mouse,”
from the following website: http://read.gov/aesop/index.html
2. A theme is listed at the bottom of each fable. Cut out the theme from each fable. (Keep “The Lion and
the Mouse” separate to use as an example.)
3. Ask a family member to shuffle the remaining themes and give you a stack of stories and a stack of
themes.

Work through the example with a family member.


1. Read the fable “The Lion and the Mouse.”
2. After you have read the fable, try to figure out the theme. You can usually determine the theme by
answering the following questions:
a. How did the main character change through the challenges he or she faced in the fable?
b. What is the lesson the main character learned by the end of the fable?
3. Write the theme on the printed copy of the fable.
4. Go back and underline the specific detail(s) from the fable that helped you decide on the theme.
5. Now read the theme on the strip of paper cut from the fable: “A kindness is never wasted.”
6. With your family member, discuss how the theme you wrote and the theme on the strip of paper are
similar or different. If the themes are different, discuss why you and the author decided on different
themes.

Now try it on your own.


1. Read the first fable in the stack provided to you.
2. After you have read the fable, try to figure out the theme.
3. Write the theme on the printed copy of the fable.
4. Underline the specific detail(s) from the fable that helped you decide on the theme.
5. Continue steps one through four with the remaining stories.

Play a theme matching game.


Using your themes, you can play a matching game with the themes that were cut from the bottom of
the fables.
1. Look through the stack of themes that were cut from the bottom of the fables. Find the theme you
believe is the best match for the theme you’ve written for each fable.
2. After you’ve matched all the themes, ask your family member to confirm the theme for each of the
fables by going back online and checking the themes at the bottom of each story.
3. With your family member, discuss how your theme and the theme on the paper are similar or different.
If they are different, discuss why you and the author came up with different themes.

Georgia Milestones Grade 4 EOG Study/Resource Guide for Students and Parents Page 91 of 152
Copyright © 2020 by Georgia Department of Education. All rights reserved.
English Language Arts (ELA)

Make more connections.


1. Do any of the fables have a similar theme to the example? If so, how are the themes similar?
2. Do any of the other fables have similar themes? If so, how are the themes similar?
3. How are the patterns of the events in the fables similar or different?
4. How are the characters’ thoughts, words, or actions in the fables similar or different?

Page 92 of 152 Georgia Milestones Grade 4 EOG Study/Resource Guide for Students and Parents
Copyright © 2020 by Georgia Department of Education. All rights reserved.
English Language Arts (ELA)

ACTIVITY
The following activity develops skills in Unit 4: Language.
Standard: ELAGSE4L5c

Preparation: Number 40 simple note cards on one side from 1 to 40.


This activity is based on the game Concentration. Work with a friend or family member to think of 20 words
and each word’s synonym, for a total of 40 words. Shuffle the cards, and lay them out on a table, number-
side down. Choose two cards at random. On one card, write the word. On the other card, write its synonym.
Do not look at the numbered sides, and set aside those two cards. Continue until all cards are completed.
Shuffle the cards when you are done.

Examples: Words and Synonyms

1. destroy 13. ruin


2. eat 18. consume
3. explore 24. investigate
4. protect 32. defend

Arrange the cards on a table in five rows of eight, with the numbers up, from 1 to 40.
Pick two cards to be turned over. If the words on the cards do not match as synonyms, the cards must be
turned back over. Now, the other person gets a turn. Whenever a match is found, the person who finds it
gets a point and the matched pair is removed from the table.

Variation:
After the cards have been created, work independently to find the matches.

Georgia Milestones Grade 4 EOG Study/Resource Guide for Students and Parents Page 93 of 152
Copyright © 2020 by Georgia Department of Education. All rights reserved.
Mathematics

MATHEMATICS

DESCRIPTION OF TEST FORMAT AND ORGANIZATION


The Grade 4 Mathematics EOG assessment consists of a total of 55 items.
You will answer a variety of item types on the test. Some of the items are selected‑response (multiple-
choice), which means you choose the correct answer from four choices. Some items are technology-
enhanced items, which means you will be asked to create a response.
The test will be given in two sections.
•• You will have up to 65 minutes per section to complete Sections 1 and 2.
•• The test will take about 60 to 130 minutes.

CONTENT
The Grade 4 Mathematics EOG assessment will measure the Grade 4 standards that are described at
www.georgiastandards.org.
The content of the assessment covers standards that are reported under these domains:
•• Operations and Algebraic Thinking
•• Number and Operations in Base 10
•• Number and Operations—Fractions
•• Measurement and Data
•• Geometry

ITEM TYPES
The Mathematics portion of the Grade 4 EOG assessment consists of selected-response (multiple-choice)
and technology-enhanced items.

Page 94 of 152 Georgia Milestones Grade 4 EOG Study/Resource Guide for Students and Parents
Copyright © 2020 by Georgia Department of Education. All rights reserved.
Mathematics

MATHEMATICS DEPTH OF KNOWLEDGE EXAMPLE ITEMS


Example items that represent applicable DOK levels are provided for you on the following pages. The items
and explanations of what is expected of you to answer them will help you prepare for the test.
All example and sample items contained in this guide are the property of the Georgia Department of
Education.

Example Item 1
Selected-Response
DOK Level 1: This is a DOK level 1 item because it assesses recall of a vocabulary term and its definition.
Mathematics Grade 4 Content Domain: Geometry
Standard: MGSE4.G.1. Draw points, lines, line segments, rays, angles (right, acute, obtuse), and
perpendicular and parallel lines. Identify these in two-dimensional figures.

Which of these figures BEST models a ray?

A.

B.

C.

D.

Correct Answer: A
Explanation of Correct Answer: The correct answer is choice (A). A ray is a part of a line with a starting
point, but no ending point. Choice (B) is incorrect because it is a line segment, a part of a line with starting
and ending points. Choice (C) is incorrect because it is a line; it has neither a starting nor an ending point.
Choice (D) is incorrect because it is an acute angle, formed by two rays.

Georgia Milestones Grade 4 EOG Study/Resource Guide for Students and Parents Page 95 of 152
Copyright © 2020 by Georgia Department of Education. All rights reserved.
Mathematics

Example Item 2
Keypad-Input Technology-Enhanced
DOK Level 2: This is a DOK level 2 item because it requires students to add fractions to solve a contextual
problem.
Mathematics Grade 4 Content Domain: Number and Operations – Fractions

a 1
Standard: MGSE4.NF.3. Understand a fraction __ ​​    ​​  with a numerator > 1 as a sum of unit fractions __ ​​   ​​  .
b b
a. Understand addition and subtraction of fractions as joining and separating parts referring to the
same whole.
b. Decompose a fraction into a sum of fractions with the same denominator in more than one way,
recording each decomposition by an equation. Justify decompositions, e.g., by using a visual
fraction model.
3 1 __ 1 1 __ 3 1 __ 2 1 1 8 __ 8 1
Examples: __
​​   ​​  = __
​​   ​​  + ​​   ​​  + __
​​   ​​  ; ​​   ​​  = __
​​   ​​  + ​​   ​​  ; 2 __ ​​   ​​  = 1 + 1 + __
​​   ​​  = __
​​   ​​  + ​​   ​​  + __
​​   ​​ . 
8 8 8 8 8 8 8 8 8 8 8 8
c. Add and subtract mixed numbers with like denominators, e.g., by replacing each mixed number
with an equivalent fraction, and/or by using properties of operations and the relationship between
addition and subtraction.
d. Solve word problems involving addition and subtraction of fractions referring to the same whole
and having like denominators, e.g., by using visual fraction models and equations to represent the
problem.

ÂÂ
Use a mouse, touchpad, or touchscreen to enter a reponse.

Page 96 of 152 Georgia Milestones Grade 4 EOG Study/Resource Guide for Students and Parents
Copyright © 2020 by Georgia Department of Education. All rights reserved.
Mathematics

Example Item 2. Continued.


Scoring Rubric

Points Description

1 The student correctly answers the question.

0 The student does not correctly answer the question.

Exemplar Response
The correct response is shown below.

2 1 2 1
This is the correct response because the context is adding __
​​   ​​  and __
​​   ​​.   The fractions __
​​   ​​  and __
​​   ​​  both refer to
4 4 4 4
the same whole. Therefore, the individual parts can be combined by adding 2 and 1 to reach 3 parts out of
3
4, which is __
​​   ​​.  
4

Georgia Milestones Grade 4 EOG Study/Resource Guide for Students and Parents Page 97 of 152
Copyright © 2020 by Georgia Department of Education. All rights reserved.
Mathematics

Example Item 3
Keypad-Input Multi-Part Technology-Enhanced
DOK Level 3: This is a DOK level 3 item because it requires students to create an expression to represent
context and to perform multiple steps in solving a contextual problem.
Mathematics Grade 4 Content Domain: Numbers and Operations in Base 10
Standard: MGSE4.NBT.5. Multiply a whole number of up to four digits by a one-digit whole number, and
multiply two two-digit numbers, using strategies based on place value and the properties of operations.
Illustrate and explain the calculation by using equations, rectangular arrays, and/or area models.
Part A

ÂÂ
Use a mouse, touchpad, or touchscreen to enter a response.

Go on to the next page to finish example item 3.

Page 98 of 152 Georgia Milestones Grade 4 EOG Study/Resource Guide for Students and Parents
Copyright © 2020 by Georgia Department of Education. All rights reserved.
Mathematics

Example Item 3. Continued.


Part B

ÂÂ
Use a mouse, touchpad, or touchscreen to enter a response.

Georgia Milestones Grade 4 EOG Study/Resource Guide for Students and Parents Page 99 of 152
Copyright © 2020 by Georgia Department of Education. All rights reserved.
Mathematics

Example Item 3. Continued.


Scoring Rubric

Points Description
2 The student correctly answers both Part A and Part B.
1 The student correctly answers either Part A OR Part B.
0 The student does not correctly answer either part.

Exemplar Response
Part A

The correct response is shown below.

This is the correct response because the context is multiplying 15 boxes times 12 phones in each box to
find the total number of phones.

Part B

The correct response is shown below.

This is the correct response because the number of boxes is 15 × 3 and the number of phones in each box
is 24. This leads to 45 boxes of 24 phones, and 45 times 24 is 1,080.

Page 100 of 152 Georgia Milestones Grade 4 EOG Study/Resource Guide for Students and Parents
Copyright © 2020 by Georgia Department of Education. All rights reserved.
Mathematics

MATHEMATICS CONTENT DESCRIPTION AND ADDITIONAL SAMPLE ITEMS


In this section, you will find information about what to study in order to prepare for the Grade 4
Mathematics EOG test. This includes key terms and important vocabulary words. This section also contains
practice questions, explanations of the correct answers, and activities that you can do on your own or with
your classmates or family to prepare for the test.
The organization of Mathematics units in this guide is based on Frameworks developed by the Curriculum
and Instructional Division of the Georgia Department of Education. These Frameworks can be accessed at
https://www.georgiastandards.org/Georgia-Standards/Pages/Math-K-5.aspx.
All example and sample items contained in this guide are the property of the Georgia Department of
Education.

CONTENT DESCRIPTION
•• Perform multi-digit multiplication and develop an understanding of dividing to find quotients involving
multi-digit dividends
•• Develop an understanding of fractions
•• Multiply fractions by whole numbers
•• Generate and analyze patterns
•• Analyze and classify geometric figures based on their properties
•• Represent and interpret data
•• Understand concepts of angles and measure angles

Georgia Milestones Grade 4 EOG Study/Resource Guide for Students and Parents Page 101 of 152
Copyright © 2020 by Georgia Department of Education. All rights reserved.
Mathematics

Unit 1: Whole Numbers, Place Value, and Rounding in Computation

In this unit, you will work with the place value system. You will round, compare, and estimate numbers. You
will use word problems with more than one step and write equations with unknown numbers.

KEY TERMS
Algorithm: A set of steps used to solve problems involving more than one step. Algorithms use the four
operations to solve word problems by using a letter, such as x, to represent an unknown number. (OA.3)
Estimate: A value that is close enough to the right answer. Solutions to multi-step word problems can be
checked to make sure that are reasonable by comparing the solution to the estimate. The estimate is
often found by rounding the numbers in the equation before solving. (OA.3)
Place value: The numerical value of a digit in a number based on its location. A digit in the tens place of a
number is 10 times the value of the same digit in the ones place. A digit in the hundreds place is 10 times
the value of the same digit in the tens place. (NBT.1)
Forms of a number: Numbers can be written in different forms using the place value of each digit. Use the
number 183 as an example.

•• Standard form: The number is written as a group of digits. For example, the standard form is 183.
•• Word form: The number is written in words. For example, the word form is one hundred eighty-three.
•• Expanded form: The number is written as the sum of the values of its digits. For example, the
expanded form is 100 + 80 + 3. (NBT.2)

Compare: Determine the value of two numbers written in different forms to see which has a greater value.

•• Greater than: If a number is larger in value, use the symbol >.


•• Less than: If a number is smaller in value, use the symbol <.
•• Equal to: If the numbers have the same value, use the symbol =. (NBT.2)

Rounding: A number can be rounded to the nearest number of a certain place value. For example, 295
can be rounded to the nearest hundred to get 300, as the number in the tens place is greater than 5.
(NBT.3)

Important Tips
"" Use the place value of each digit when writing numbers from word form. Remember to keep in mind
place value when writing numbers. For example, one thousand twenty-four is written as 1,024 with a 1
in the thousands place, 2 in the tens place, and 4 in the ones place.
"" When using rounded numbers in an equation, the answer will be an estimate.
"" Add and subtract whole numbers using place value to regroup as needed. When adding, a place value
that has a sum of 10 or greater will need to regroup into the higher place value. When subtracting,
find the difference between the first and second number. If a digit in the first number is smaller than
the digit in the same place in the second number, regroup from a higher place value into a lower place
value.

Page 102 of 152 Georgia Milestones Grade 4 EOG Study/Resource Guide for Students and Parents
Copyright © 2020 by Georgia Department of Education. All rights reserved.
Mathematics

Sample Items 1–4

Item 1
Selected-Response
The population of Pleasantville is 2,378.

What is the population of the city, rounded to the nearest hundred?

A. 2,000
B. 2,300
C. 2,380
D. 2,400

Item 2
Selected-Response
Subtract.

2,406 − 157

A. 2,249
B. 2,259
C. 2,349
D. 2,351

Georgia Milestones Grade 4 EOG Study/Resource Guide for Students and Parents Page 103 of 152
Copyright © 2020 by Georgia Department of Education. All rights reserved.
Mathematics

Item 3
Drag-and-Drop Technology-Enhanced

ÂÂ
Use a mouse, touchpad, or touchscreen to move a number into each blank. Each number may be
used 8 times.

Page 104 of 152 Georgia Milestones Grade 4 EOG Study/Resource Guide for Students and Parents
Copyright © 2020 by Georgia Department of Education. All rights reserved.
Mathematics

Item 4
Drop-Down Technology-Enhanced

ÂÂ
Use a mouse, touchpad, or touchscreen to click the arrow beside each of the two blank boxes. When
you click the arrow, a drop-down menu will appear, showing you all the possible options for that blank
box. Each drop-down menu with its options is shown below.

Georgia Milestones Grade 4 EOG Study/Resource Guide for Students and Parents Page 105 of 152
Copyright © 2020 by Georgia Department of Education. All rights reserved.
Mathematics

Unit 2: Multiplication and Division of Whole Numbers

In this unit, you will use multiplication, division, and word problems with more than one step. You will use
the properties of operations. You will work with prime and composite numbers and patterns.

KEY TERMS
Multiplication: The operation used to find the total number of objects in a set of equal groups. For
example, 3 groups of 4 objects have a total of 12 objects.
Multiplier: The number that is being multiplied by. For example, in 2 × 3 = 6, the multiplier is 2.
Multiplicand: The number that is getting multiplied. For example, in 2 × 3 = 6, the multiplicand is 3.
Product: The answer of a multiplication problem. For example, in 2 × 3 = 6, the product is 6.
Division: The operation used to partition or break apart the total number of objects into a number of
groups or into groups of a specific size. For example, 15 objects divided into 3 groups have 5 objects in
each group, or 15 objects divided into groups of 3 will create 5 groups.
Dividend: The number that is divided. For example, in 20 ÷ 4 = 5, the dividend is 20.
Divisor: The number that is divided by. For example, in 20 ÷ 4 = 5, the divisor is 4.
Quotient: The answer to a division problem. For example, in 20 ÷ 4 = 5, the quotient is 5.
Remainder: The amount remaining when division occurs. For example, when 7 cookies are shared among
3 people, each person will get 2 cookies and 1 will be left over. The remainder is 1.
Multiplicative comparison: Comparing the value of one object to the value of another, using phrases such
as “3 times as long.” (OA.1)
Equation: A grouping of numbers, letters, and operations with an equal sign.
Expression: A grouping of numbers, letters, and operations without an equal sign.
Unknown: A value in an equation or expression that is missing. The unknown value can be represented by
a symbol, letter, empty box, or even a question mark. When an unknown value is represented by a letter,
for example x, it is called a variable.

Properties of Operations
•• Commutative Property: Numbers can be multiplied in any order and the product will stay the same.
•• Associative Property: Three or more factors can be grouped together in any way and the product will
stay the same.
•• Distributive Property: Multiplication can be distributed over addition. For example,
2(3 + 4) = 2 × 3 + 2 × 4. (NBT.5)
Factors: Numbers that multiply together to equal the given number. For example, 4 and 2 are factors of 8;
4 × 2 = 8. (OA.4)
Multiple: The product of the given number and another number. For example, 12 is a multiple of 3
because 3 × 4 = 12. (OA.4)
Prime: A number that can be broken down into factors of only 1 and itself. (OA.4)
Composite: A number that has more factors than 1 and itself. (OA.4)
Patterns: Repeated sequences of numbers or shapes that follow a set of rules, such as “add 5.” (OA.5)

Page 106 of 152 Georgia Milestones Grade 4 EOG Study/Resource Guide for Students and Parents
Copyright © 2020 by Georgia Department of Education. All rights reserved.
Mathematics

Important Tips
"" When listing multiples of a number, include the given number. The smallest multiple of a number is the
number itself. For example, 5 is a multiple of 5 using the expression 5 × 1.
"" The number of factors a number has is not related to how small or large the number is. A number with
a greater value may not have a large number of factors.
"" A prime number has factors of only one and itself. Two is the smallest prime number. A composite
numbers is a number that has factors other than one and itself.
"" Notice that 1 is neither prime nor composite.
"" Solve word problems involving multiplicative comparison by creating a drawing or equation to represent
the problem. A letter can be used in an equation for an unknown number. Use multiplication or division
to solve for the unknown number.
"" Use place value and properties of operations to multiply and divide whole numbers. Use models such
as arrays, area models, and equations to illustrate the problem.

Georgia Milestones Grade 4 EOG Study/Resource Guide for Students and Parents Page 107 of 152
Copyright © 2020 by Georgia Department of Education. All rights reserved.
Mathematics

Sample Items 5–8

Item 5
Selected-Response
Multiply.

35 × 43

A. 425
B. 1,325
C. 1,405
D. 1,505

Item 6
Selected-Response
There are 3 times as many red crayons in a bucket as blue crayons. There are 8 blue crayons.

Which equation represents the number of red crayons in the bucket?

A. 16 ÷ 8 = 3
B. 8–3=5
C. 3 + 8 = 11
D. 3 × 8 = 24

Page 108 of 152 Georgia Milestones Grade 4 EOG Study/Resource Guide for Students and Parents
Copyright © 2020 by Georgia Department of Education. All rights reserved.
Mathematics

Item 7
Multi-Part Multi-Select Technology-Enhanced
Part A
A factor pair of 93 is 1 and 93.

What is another factor pair of 93?

A. 3 and 9
B. 3 and 31
C. 9 and 10
D. 3 and 90

Part B
Select TWO numbers that are multiples of 8.

A. 8
B. 22
C. 56
D. 68
E. 84

Georgia Milestones Grade 4 EOG Study/Resource Guide for Students and Parents Page 109 of 152
Copyright © 2020 by Georgia Department of Education. All rights reserved.
Mathematics

Item 8
Drag-and-Drop Technology-Enhanced

ÂÂ
Use a mouse, touchpad, or touchscreen to move the numbers into the diagram. Each number may be
used 1 time.

Page 110 of 152 Georgia Milestones Grade 4 EOG Study/Resource Guide for Students and Parents
Copyright © 2020 by Georgia Department of Education. All rights reserved.
Mathematics

Unit 3: Fraction Equivalents

In this unit, you will work with fractions, including improper and equivalent fractions and mixed numbers.
You will compare fractions and create common denominators and numerators.

KEY TERMS
Fraction: A number used to represent equal parts of a whole. (NF.1)
2
Numerator: The top number in a fraction. For example, in the fraction __
​​   ​​ , the 2 is the numerator. (NF.1)
3
2
Denominator: The bottom number in a fraction. For example, in the fraction __ ​​   ​​ , the 3 is the denominator.
(NF.1) 3

Improper fraction: A fraction with a value greater than 1. This means the numerator is greater than the
9
denominator. For example, __ ​​   ​​  is an improper fraction. (NF.1)
2
Mixed number: A number that includes a whole number and a fraction, and it has an improper fraction
9 1
that it is equal to. For example, __ ​​   ​​  is an improper fraction, and 4 ​​ __ ​​  is the equivalent mixed number. (NF.1)
2 2
Whole number: Counting numbers are whole numbers. For example, 0, 1, 2, 3, and so are on are all whole
numbers. (NF.1)
Equivalent fractions: Fractions that have the same value or represent the same point on the number
line. (NF.1) Equivalent fractions are created by multiplying the numerator and denominator by the same
(1 × 4) __ 4 4
number, which is the same as multiplying the fraction by 1. For example, _______ ​​  = ​​   ​​ , so __
 ​​  ​​   ​​  is equivalent
1 (2 × 4) 8 8
to __
​​   ​​ .  The fraction now describes a different number of parts in the whole and the parts are a different size
2
than they were, but the value of the fraction remains the same. (NF.1)
Compare: Determine the value of each of two fractions to see which fraction is larger. Fractions can be
compared when the whole represented by each fraction is divided into equal-size parts and the wholes are
the same size.
Greater than: When the first fraction is larger in value than the second fraction, use the symbol >.
Less than: When the first fraction is smaller in value than the second fraction, use the symbol <.
Equal to: When the fractions have the same value (equivalent fractions), use the symbol =. (NF.2)
Common denominator: Using strategies for creating equivalent fractions, create fractions that have the
same denominator. This makes comparing the fractions easier. (NF.2)
Common numerator: Using strategies for creating equivalent fractions, create fractions that have the
same numerator. This makes comparing the fractions easier. (NF.2)
Benchmark fraction: Common fraction used to compare other fractions, such as ​​ 1 __ 2 ​​. (NF.2)

Important Tips
"" When comparing fractions, use both the numerator and the denominator to find the value of the
fraction. The numerator tells the number of parts out of the whole, and the denominator tells how
many parts are in the whole.
"" Fractions in a comparison must represent parts of wholes that are the same size. When using models
to compare fractions, use models that are the same size and shape.

Georgia Milestones Grade 4 EOG Study/Resource Guide for Students and Parents Page 111 of 152
Copyright © 2020 by Georgia Department of Education. All rights reserved.
Mathematics

Sample Items 9–11

Item 9
Selected-Response
Look at the model.

Which fraction is equivalent to the shaded part of this model?


1
A. __
​​   ​​  
4
1
B. __ ​​   ​​  
3
1
C. __ ​​   ​​  
2
1
D. __ ​​   ​​  
6

Item 10
Selected-Response
Look at the expression.
1
> __
​​   ​​  
2
Which fraction goes in the to make this expression TRUE?
1
A. __
​​   ​​  
4
2
B. __ ​​   ​​  
3
2
C. __ ​​   ​​  
4
1
D. __ ​​   ​​  
3

Page 112 of 152 Georgia Milestones Grade 4 EOG Study/Resource Guide for Students and Parents
Copyright © 2020 by Georgia Department of Education. All rights reserved.
Mathematics

Item 11
Number-Line Technology-Enhanced

ÂÂ
Use a mouse, touchpad, or touchscreen to plot points on the number line. At most 1 point can be
plotted.

Georgia Milestones Grade 4 EOG Study/Resource Guide for Students and Parents Page 113 of 152
Copyright © 2020 by Georgia Department of Education. All rights reserved.
Mathematics

Unit 4: Operations with Fractions

In this unit, you will add, subtract, and multiply fractions. You will continue to work with improper fractions
and mixed numbers.

KEY TERMS
Proper fraction: A fraction where the value of the numerator is less than the value of the denominator.
Unit fraction: A proper fraction that has a numerator of 1.
Common denominator: When fractions have the same denominator, they can be added or subtracted.
(NF.3)
Unlike denominators: Fractions that do not have the same denominator cannot be added or subtracted
until the fractions are replaced with equivalent fractions so that they have a common denominator.
Equivalent: Same value. Equivalent fractions have the same value and represent the same point on a
number line.
Improper fractions: Fractions that have a numerator that is greater than the denominator. An improper
fraction always has a value greater than 1. (NF.3)
Mixed numbers: A whole number and a fraction. Mixed numbers are another way to write an improper
fraction, and they always have a value greater than 1. (NF.3)

Important Tips
"" Fractions in an equation must represent parts of wholes that are the same size. When using models to
solve the equations, use models that are the same size and shape.
"" Decompose a fraction by separating the given fraction into a sum of smaller fractions.
3 1 __ 2
For example, __
​​   ​​   = __
​​   ​​  + ​​   ​​ . 
5 5 5
2 1
"" A fraction is a multiple of a unit fraction. For example, __
​​   ​​  is 2 × __
​​   ​​.   This strategy can be used to multiply
3 3
a fraction by a whole number.

"" Multiplying a fraction by a whole number is the same as repeatedly adding the fraction.
1 1 1 __ 1
For example, __
​​   ​​   × 3 = __
​​   ​​  + __
​​   ​​  + ​​   ​​  .
4 4 4 4
"" Word problems can be represented using an equation or a fraction model.

Page 114 of 152 Georgia Milestones Grade 4 EOG Study/Resource Guide for Students and Parents
Copyright © 2020 by Georgia Department of Education. All rights reserved.
Mathematics

Sample Items 12–15

Item 12
Selected-Response
Which equation is TRUE?
2 1 __ 1
A. __
​​   ​​  = __ ​​   ​​  + ​​   ​​  
8 5 3
3 1 __ 1 1
B. __ ​​   ​​  = __ ​​   ​​  + ​​   ​​  + __ ​​   ​​  
8 3 3 3
6 1 __ 1 1 __ 1 1 __ 1
C. __ ​​   ​​  = __ ​​   ​​  + ​​   ​​  + __ ​​   ​​  + ​​   ​​  + __ ​​   ​​  + ​​   ​​  
8 8 8 8 8 8 8
6 1 __ 1 1 __ 1 1 __ 1 1 __ 1
D. __ ​​   ​​  = __ ​​   ​​  + ​​   ​​  + __ ​​   ​​  + ​​   ​​  + __ ​​   ​​  + ​​   ​​  + __
​​   ​​  + ​​   ​​  
8 6 6 6 6 6 6 6 6

Item 13
Selected-Response
1
Sarita has 3 rolls of ribbon. Sarita cuts off __
​​   ​​  yard from each roll.
2

How much ribbon does she cut off in all?

A. 1 yard

1
B. 1 ​​ __ ​​  yards
2
C. 3 yards

1
D. 3 ​​ __ ​​  yards
2

Georgia Milestones Grade 4 EOG Study/Resource Guide for Students and Parents Page 115 of 152
Copyright © 2020 by Georgia Department of Education. All rights reserved.
Mathematics

Item 14
Multi-Select Technology-Enhanced
4
Yolanda has __
​​   ​​  of a bag of dog food remaining. She will put the remaining dog food in smaller bags.
5

Select THREE equations that can represent fractions of the bag of dog food that Yolanda can put in
smaller bags.
4 1 __ 1 1 __ 1
A. __
​​   ​​  = __ ​​   ​​  + ​​   ​​  + __ ​​   ​​  + ​​   ​​  
5 5 5 5 5
4 2 __ 2
B. __ ​​   ​​  = __ ​​   ​​  + ​​   ​​  
5 3 2
4 4 __ 1
C. __ ​​   ​​  = __ ​​   ​​  + ​​   ​​  
5 1 5
4 3 __ 1
D. __ ​​   ​​  = __ ​​   ​​  + ​​   ​​ 
5 5 5
4 2 __ 1 1
E. __ ​​   ​​  = __ ​​   ​​  + ​​   ​​  + __ ​​   ​​ 
5 5 5 5
4 1 __ 4
F. __ ​​   ​​  = __ ​​   ​​  + ​​   ​​ 
5 5 5

Page 116 of 152 Georgia Milestones Grade 4 EOG Study/Resource Guide for Students and Parents
Copyright © 2020 by Georgia Department of Education. All rights reserved.
Mathematics

Item 15
Drag-and-Drop Multi-Part Technology-Enhanced
Part A

ÂÂ
Use a mouse, touchpad, or touchscreen to move the shaded square into model. The shaded square
may be used 20 times.

Go on to the next page to finish item 15.

Georgia Milestones Grade 4 EOG Study/Resource Guide for Students and Parents Page 117 of 152
Copyright © 2020 by Georgia Department of Education. All rights reserved.
Mathematics

Item 15. Continued.


Part B

ÂÂ
Use a mouse, touchpad, or touchscreen to select a response.

Page 118 of 152 Georgia Milestones Grade 4 EOG Study/Resource Guide for Students and Parents
Copyright © 2020 by Georgia Department of Education. All rights reserved.
Mathematics

Unit 5: Fractions and Decimals

In this unit, you will add and subtract fractions. You will compare decimals and work with place value.

KEY TERMS

Decimal: Another way to write a fraction. Both a decimal and a fraction show a value that is between
6
whole numbers. For example, ___
​​    ​​ or 0.6 is a value between the whole numbers 0 and 1. (NF.6)
10
Place value: The value of a digit in a number based on its location related to the decimal point. A digit in
the tenths place of a number is 10 times the value of the same digit in the hundredths place. A digit in the
1
tenths place is ___
​​    ​​ the value of the same digit in the ones place. (NF.6)
10
•• Tenths place: This is the first place to the right of the decimal point. A decimal of 0.1 would have a
1
value equivalent to ___
​​    ​​ . 
10
•• Hundredths place: This is the second place to the right of the decimal point. A decimal of 0.01 would
1
have a value equivalent to ____ ​​     ​​ . (NF.6)
100
Compare: Determine the value of two decimal numbers and determine whether one number has a greater
value or whether the numbers have an equal value.

•• Greater than: When the first number has a greater value than the other number, use the symbol >.
•• Less than: When the first number has a smaller value than the other number, use the symbol <.
•• Equal to: When both numbers have the same value, use the symbol =. (NF.7)

Important Tips
"" When comparing decimal numbers, look at the place value of each digit. The location of the digit
determines its value.
"" Fraction models and drawings can be used to compare decimals.
"" Decimals can be changed into fractions with a denominator of 10 or 100 and then used to create the
35 3 5
model. A decimal such as 0.35 can be written as ____
​​    ​​ or ___
​​    ​​ + ____
​​     ​​. 
100 10 100

Georgia Milestones Grade 4 EOG Study/Resource Guide for Students and Parents Page 119 of 152
Copyright © 2020 by Georgia Department of Education. All rights reserved.
Mathematics

Sample Items 16 and 17

Item 16
Selected-Response
3
Which fraction is equivalent to ___
​​    ​​  ?
10
3
A. ____
​​     ​​ 
100
6
B. ____
​​     ​​ 
100
10
C. ____ ​​    ​​ 
100
30
D. ____ ​​    ​​ 
100

Item 17
Selected-Response
43
Which decimal is equivalent to ____
​​    ​​  
?
100

A.  0.043
B.  0.43
C.  4.3
D. 43.00

Page 120 of 152 Georgia Milestones Grade 4 EOG Study/Resource Guide for Students and Parents
Copyright © 2020 by Georgia Department of Education. All rights reserved.
Mathematics

Unit 6: Geometry

In this unit, you will study two-dimensional figures and their properties. You will work with angles, parallel
and perpendicular lines, points, lines, line segments, rays, and lines of symmetry.

KEY TERMS
Plane figure: A two-dimensional figure that consists of length and width. Plane figures have properties,
some of which are listed here.

•• Angles:
•• Acute: An angle measure less than 90°.
•• Obtuse: An angle measure greater than 90°.
•• Right: An angle measure equal to 90°; this is noted on geometric figures by a box in a corner
that is 90°.
•• Parallel lines: Two lines that are always an equal distance apart.
•• Perpendicular lines: Two lines that intersect at a 90° angle.
•• Point: A location represented by a dot.
•• Line: A straight line that continues in each direction with no endpoints.
•• Line segment: Part of a straight line that begins and ends at two specific points.
•• Ray: A part of a straight line that continues in one direction and has one endpoint.
•• Vertex: A point where two or more line segments meet. (G.1)
Category: A group of two-dimensional figures that share at least one property. Several categories are
defined below. (G.2)

•• Polygon: A closed plane figure made of three or more line segments, called sides, that have endpoints
that intersect to form vertices.

•• Triangle: A polygon with three sides.

•• Equilateral triangle: A triangle with three sides of equal length.

Georgia Milestones Grade 4 EOG Study/Resource Guide for Students and Parents Page 121 of 152
Copyright © 2020 by Georgia Department of Education. All rights reserved.
Mathematics

•• Isosceles triangle: A triangle with two sides of equal length.

•• Scalene triangle: A triangle with three sides of different lengths.

•• Right triangle: A triangle with one right angle.

•• Quadrilateral: A polygon with four sides.

•• Trapezoid: A quadrilateral with at least one pair of parallel sides.

•• Parallelogram: A quadrilateral with two pairs of parallel sides.

•• Rhombus: A parallelogram with four sides of equal length.

Page 122 of 152 Georgia Milestones Grade 4 EOG Study/Resource Guide for Students and Parents
Copyright © 2020 by Georgia Department of Education. All rights reserved.
Mathematics

•• Rectangle: A parallelogram with four right angles.

•• Square: A rhombus that is also a rectangle with four sides of equal length and four right angles.

Line of symmetry: A line across a figure such that the figure can be folded along the line into matching
parts. (G.3)

Important Tips
"" The measure of an angle is not impacted by the length of the lines that make up the angle. When
comparing angles, use a protractor to measure the angles or visually compare the degrees of measure.
"" A right angle can be estimated using the corner of a piece of paper or book. These everyday objects
are rectangles and therefore have four right angles.

Georgia Milestones Grade 4 EOG Study/Resource Guide for Students and Parents Page 123 of 152
Copyright © 2020 by Georgia Department of Education. All rights reserved.
Mathematics

Sample Items 18–21

Item 18
Selected-Response
Which type of lines meet at a 90° angle?

A. curved
B. diagonal
C. parallel
D. perpendicular

Item 19
Selected-Response
Which figure has exactly one line of symmetry?

A.

B.

C.

D.

Page 124 of 152 Georgia Milestones Grade 4 EOG Study/Resource Guide for Students and Parents
Copyright © 2020 by Georgia Department of Education. All rights reserved.
Mathematics

Item 20
Coordinate-Graph Technology-Enhanced

ÂÂ
Use a mouse, touchpad, or touchscreen to draw line segments on the grid. At most 4 line segments
can be placed.

Georgia Milestones Grade 4 EOG Study/Resource Guide for Students and Parents Page 125 of 152
Copyright © 2020 by Georgia Department of Education. All rights reserved.
Mathematics

Item 21
Coordinate-Graph Technology-Enhanced

ÂÂ
Use a mouse, touchpad, or touchscreen to draw line segments on the grid. At most 2 line segments
and 1 point can be placed.

Page 126 of 152 Georgia Milestones Grade 4 EOG Study/Resource Guide for Students and Parents
Copyright © 2020 by Georgia Department of Education. All rights reserved.
Mathematics

Unit 7: Measurement

In this unit, you will work with different units of measurement, including time. You will record
measurements on line plots and use protractors to measure angles. You will determine the area and
perimeter of rectangles.

KEY TERMS
Conversion: Changing between units within the same measurement system. For example, 12 inches is
equal to 1 foot, so to convert 3 feet to inches, you would multiply 12 by 3 to get 36 inches. (MD.1)
Units of measurement: The following lists the units used for different types of measurements.
Customary Measurement

•• Liquid volume: cups, pints, quarts, and gallons


•• Length: inches, feet, yards, and miles
•• Weight: ounces, pounds, and tons

Metric Measurement

•• Liquid volume: milliliters and liters


•• Length: centimeters, meters, and kilometers
•• Mass: grams and kilograms

Time: seconds, minutes, and hours (MD.1)


Area: The amount of space inside a two-dimensional shape. The formula for finding the area of a rectangle
is A = length × width. (MD.3)
Perimeter: The distance around a two-dimensional shape. The formula for finding the perimeter of a
rectangle is P = 2(length + width). (MD.3)

Line plot: A way of recording measurements for a group of objects. These measurements can include
liquid volume, length, mass, and time. For example, a line is marked with measurements using unit
1 1 1
fractions, including __
​​   ​​ ,  __
​​   ​​ ,  and __
​​   ​​.   Marks are placed above the measurements on the line. The line plot is
8 4 2
used to answer questions about the measurements shown. (MD.4)

Angles: Two rays that have the same endpoint form an angle. They are measured as part of a circle with
the endpoint as the center. The measure of an angle is the part between the two rays. (MD.5) Angles are
measured in degrees using a protractor. (MD.6)

Georgia Milestones Grade 4 EOG Study/Resource Guide for Students and Parents Page 127 of 152
Copyright © 2020 by Georgia Department of Education. All rights reserved.
Mathematics

Important Tips
"" To convert a measurement, such as yards, choose another unit used to measure length within the
same measurement system, such as feet or inches.
"" Estimate the size of an angle as greater than or less than 90° before measuring with a protractor.
If the estimate of the angle is less than 90°, then use the smaller number on the protractor. If the
estimate is greater than 90°, then use the larger number on the protractor.
"" Use the four operations to solve word problems involving liquid volume, mass, intervals of time, and
money within the same units of measure. If the units of measure are not the same, convert larger units
into smaller units, such as feet into inches. These word problems may include decimals or fractions.
"" An angle can be divided into smaller angles that do not overlap. The measure of non-overlapping
parts can be added together to find the measure of the whole angle. You can also find the measure of
unknown angles by writing an equation with a letter for the unknown angle measure.

Page 128 of 152 Georgia Milestones Grade 4 EOG Study/Resource Guide for Students and Parents
Copyright © 2020 by Georgia Department of Education. All rights reserved.
Mathematics

Sample Items 22–27

Item 22
Selected-Response
Ms. Johnson planted a rectangular garden. The length of the garden is 8 feet. The width is 7 feet.

What is the perimeter of the garden?

Perimeter = 2(length + width)

A. 15 feet
B. 20 feet
C. 30 feet
D. 56 feet

Item 23
Selected-Response
Look at the angle measures in the right angle.

75°
n

What is the measure of the unknown angle, n?

A.  15°
B.  25°
C.  90°
D. 180°

Georgia Milestones Grade 4 EOG Study/Resource Guide for Students and Parents Page 129 of 152
Copyright © 2020 by Georgia Department of Education. All rights reserved.
Mathematics

Item 24
Multi-Part Technology-Enhanced
Katie buys a container that has 24 ounces of iced tea mix. This container has enough iced tea mix to
make 10 quarts of iced tea.

(1 cup = 8 ounces)
(4 cups = 1 quart)
(1 gallon = 4 quarts)
(2 cups = 1 pint)

Part A
Which quantity is equivalent to 10 quarts?

A. 2 gallons
B. 3 pints
C. 32 ounces
D. 40 cups

Part B
1
Katie used 1​​ __ ​​  tablespoons of iced tea mix for every cup of water. She used 6 cups of water to make
2
some iced tea.

Which statement describes the number of tablespoons of iced tea mix and the number of ounces of
water Katie used?

A. Katie used 9 tablespoons of iced tea mix and 14 ounces of water.


B. Katie used 9 tablespoons of iced tea mix and 48 ounces of water.
C. Katie used 12 tablespoons of iced tea mix and 14 ounces of water.
D. Katie used 12 tablespoons of iced tea mix and 48 ounces of water.

Page 130 of 152 Georgia Milestones Grade 4 EOG Study/Resource Guide for Students and Parents
Copyright © 2020 by Georgia Department of Education. All rights reserved.
Mathematics

Item 25
Bar-Graph Technology-Enhanced

ÂÂ
Use a mouse, touchpad, or touchscreen to create each bar in the bar graph.

Georgia Milestones Grade 4 EOG Study/Resource Guide for Students and Parents Page 131 of 152
Copyright © 2020 by Georgia Department of Education. All rights reserved.
Mathematics

Item 26
Drop-Down Technology-Enhanced

ÂÂ
Use a mouse, touchpad, or touchscreen to click the arrow beside each of the three blank boxes.
When you click the arrow, a drop-down menu will appear, showing you all the possible options for that
blank box. Each drop-down menu with its options is shown below.

Page 132 of 152 Georgia Milestones Grade 4 EOG Study/Resource Guide for Students and Parents
Copyright © 2020 by Georgia Department of Education. All rights reserved.
Mathematics

Item 27
Line-Plot Technology-Enhanced

ÂÂ
Use a mouse, touchpad, or touchscreen to add Xs to the line plot. At most 3 Xs can be plotted for
each value.

Georgia Milestones Grade 4 EOG Study/Resource Guide for Students and Parents Page 133 of 152
Copyright © 2020 by Georgia Department of Education. All rights reserved.
Mathematics

MATHEMATICS ADDITIONAL SAMPLE ITEM KEYS

Standard/ DOK Correct


Item Explanation
Element Level Answer
The correct answer is choice (D) 2,400. To round to the
nearest hundred, the value of the digit in the tens place
is evaluated. If the digit in the tens place is greater than
5, the digit in the hundreds place rounds to the greater
1 MGSE4.NBT.3 2 D hundred. Choice (A) is incorrect because it is the result of
rounding to the nearest thousand. Choice (B) is incorrect
because it incorrectly shows rounding to the nearest
hundred. Choice (C) is incorrect because it shows rounding
to the nearest ten.
The correct answer is choice (A) 2,249. This subtraction
problem requires regrouping with a zero. Choices (B) and
2 MGSE4.NBT.4 2 A (C) are incorrect because both were regrouped incorrectly.
Choice (D) is incorrect because digits were subtracted
without regrouping.
3 MGSE4.NBT.2 2 N/A See scoring rubric and exemplar response on page 139.
4 MGSE4.NBT.2 2 N/A See scoring rubric and exemplar response on page 140.
The correct answer is choice (D) 1,505. Using an area
model, add 1200 + 200 + 90 + 15. Choice (A) is incorrect
because it is the result of multiplying 40 times 30 and
5 MGSE4.NBT.5 2 D getting 120 instead of 1,200. Choice (B) is incorrect
because it is the result of multiplying 40 times 5 and
getting 20 instead of 200. Choice (C) is incorrect because
it is the result of adding tens incorrectly.
The correct answer is choice (D) 3 × 8 = 24. This word
problem asks which equation represents the number of
red crayons. This was best shown with the operation of
6 MGSE4.OA.1 2 D
multiplication. Choice (A) is incorrect because the equation
is not true. Choices (B) and (C) are incorrect because they
use the wrong operations.
Part A: The correct answer is choice (B) 3 and 31.
Choice (A) is incorrect because 3 and 9 are the digits of
93 but 9 is not a factor of 93. Choice (C) is incorrect
Part A: because neither 9 nor 10 is a factor of 93 but factors of
B 90. Choice (D) is incorrect because 3 + 90 is 93 but 90 is
7 MGSE4.OA.4 2 not a factor of 93.
Part B:
A, C Part B: The correct answers are choices (A) and (C).
Choices (B), (D), and (E) are incorrect because there is no
whole number that can be multiplied by 8 that would result
in these numbers.

8 MGSE4.OA.4 2 N/A See scoring rubric and exemplar response on page 141.

Page 134 of 152 Georgia Milestones Grade 4 EOG Study/Resource Guide for Students and Parents
Copyright © 2020 by Georgia Department of Education. All rights reserved.
Mathematics

Standard/ DOK Correct


Item Explanation
Element Level Answer
1
The correct answer is choice (B) __
​​   ​​.   The circle is divided
3
into 12 equal parts, and 4 of them are shaded. Four out
1
of 12 is equivalent to __
​​   ​​.   Choice (A) is incorrect because
3
9 MGSE4.NF.1 1 B it is equivalent to 3 out of 12 parts shaded. Choice (C)

is incorrect because it is equivalent to 6 out of 12 parts

shaded. Choice (D) is incorrect because it is equivalent to

2 out of 12 parts shaded.


2
The correct answer is choice (B) __
​​   ​​.   Rewriting both fractions
3
2 4 1 3
with a common denominator gives __ ​​   ​​   = __
​​   ​​  and __
​​   ​​  = __
​​   ​​.  
3 6 2 6
10 MGSE4.NF.2 1 B 4 3 2 1
Since __ ​​   ​​  > __
​​   ​​  then __
​​   ​​  > __​​   ​​.   Choice (A) is incorrect because
6 6 3 2
__ 1 1 2 1
​​   ​​  < __
​​   ​​.   Choice (C) is incorrect because __ ​​   ​​  = __
​​   ​​.   Choice (D) is
4 2 4 2
1 1
incorrect because __ ​​   ​​  < __​​   ​​.  
3 2

11 MGSE4.NF.2 3 N/A See scoring rubric and exemplar response on page 142.

6
The correct answer is choice (C); __
​​   ​​  can be made by joining
8
6 unit fractions of the same denominator. Choice (A) is

incorrect because the sum of the unit fractions equals


12 MGSE4.NF.3a 2 C 8 2
___
​​    ​​,  not __
​​   ​​.  
Choice (B) is incorrect because the sum of the
15 8
3 3
unit fractions equals __
​​   ​​  = 1, not __
​​   ​​.   Choice (D) is incorrect
3 8
8 6
because the sum of the unit fractions equals __ ​​   ​​ ,  not __
​​   ​​.  
6 8

Georgia Milestones Grade 4 EOG Study/Resource Guide for Students and Parents Page 135 of 152
Copyright © 2020 by Georgia Department of Education. All rights reserved.
Mathematics

Standard/ DOK Correct


Item Explanation
Element Level Answer
1
The correct answer is choice (B) 1 __ ​​   ​​  yards. This is
2
1 __ 1 1 3
the same as ​​   ​​   + ​​   ​​    + ​​   ​​ ,  which equals __
__ __ ​​   ​​.   Two pieces
2 2 2 2
1
__
of ribbon that are ​​   ​​  yard equal 1 yard in total plus an
2
__ 1
additional ​​   ​​    yard. Choice (A) is incorrect because it is the
13 MGSE4.NF.4c 2 B 2
total amount cut off only 2 rolls. Choice (C) is incorrect

because it is the number of pieces of ribbon. Choice (D) is

incorrect because it is the sum of two numbers given in the

problem.

The correct answers are choices (A), (D), and (E). Each
of these recognizes that when adding fractions, the
denominator must be the same and the numerators
14 MGSE4.NF.3b 2 A/D/E are added together. Choice (B) is incorrect because the
denominators are added. Choice (C) is incorrect because
it incorrectly creates unit fractions. Choice (F) is incorrect
5
because it results in __
​​   ​​  = 1.
5
15 MGSE4.NF.4 2 N/A See scoring rubric and exemplar response on page 143.
30 3
The correct answer is choice (D) ____
​​    ​​. The fraction ___
​​    ​​ 
100 10
30
____
has the same value as ​​    ​​ since 3 times 10 equals 30
16 MGSE4.NF.5 1 D 100
and 10 times 10 equals 100. Choices (A), (B), and (C) are
3
incorrect because the fractions are not equivalent to ___
​​    ​​. 
10
The correct answer is choice (B) 0.43. The decimal

0.43 means there are 43 hundredths; this is equivalent


43
to ____
​​    ​​. Choice (A) is incorrect because 0.043 means
17 MGSE4.NF.6 2 B 100
43
43 thousandths, or ______ ​​    ​​. Choice (C) is incorrect because
1000
3
4.3 means 4 wholes and 3 tenths, or 4 ___ ​​    ​​.  Choice (D) is
10
incorrect because 43.00 means 43 wholes.

Page 136 of 152 Georgia Milestones Grade 4 EOG Study/Resource Guide for Students and Parents
Copyright © 2020 by Georgia Department of Education. All rights reserved.
Mathematics

Standard/ DOK Correct


Item Explanation
Element Level Answer
The correct answer is choice (D) perpendicular.
Perpendicular lines intersect at a right angle, or
90 degrees. Choice (A) is incorrect because curved
lines don’t meet at an angle; an angle is formed by the
18 MGSE4.G.1 1 D intersection of two lines, segments, or rays. Choice (B)
is incorrect because not all diagonal lines intersect.
Choice (C) is incorrect because parallel lines are lines that
will never intersect; they will always be the same distance
apart from one another.
The correct answer is choice (D). An isosceles triangle
has exactly one line of symmetry. Choice (A) is incorrect
because a square has four lines of symmetry. Choice (B)
19 MGSE4.G.3 2 D
is incorrect because the figure has no lines of symmetry.
Choice (C) is incorrect because a circle has an unlimited
number of lines of symmetry.
20 MGSE4.G.3 2 N/A See scoring rubric and exemplar response on page 144.
21 MGSE4.G.3 2 N/A See scoring rubric and exemplar response on page 145.
The correct answer is choice (C) 30 feet. The perimeter is
found by adding all side lengths of a figure. Choice (A) is
incorrect because just two sides of the figure were added.
22 MGSE4.MD.3 2 C
Choice (B) is incorrect because it is the result of incorrectly
adding 16 and 14 and getting 20. Choice (D) is incorrect
because it is the area, the space inside the figure.
The correct answer is choice (A) 15°. The two smaller
angles together form a right angle, so their sum must be
90°. Choice (B) is incorrect because a right angle does
23 MGSE4.MD.7 2 A
not measure 100°. Choice (C) is incorrect because 90° is
the entire measurement of the right angle. Choice (D) is
incorrect because it is the measurement of a straight line.
Part A: The correct answer is choice (D) 40 cups.
Ten quarts is equivalent to 40 cups. Choice (A) is incorrect
because 10 quarts is equivalent to 2.5 gallons. Choice (B)
is incorrect because 10 quarts is equivalent to 20 pints.
Choice (C) is incorrect because 10 quarts is equivalent to
320 ounces.
Part A:
D Part B: The correct answer is choice (B). Six cups of water
24 MGSE4.MD.1 3 equals 48 ounces of water, and 6 whole tablespoons plus
Part B:
6 half tablespoons of ice tea mix is 9 tablespoons.
B
Choice (A) is incorrect because it adds 8 and 6 to get
14 ounces. Choice (C) is incorrect because it incorrectly
found the number of tablespoons and the number of
ounces of water Katie used. Choice (D) is incorrect
because it incorrectly found the number of tablespoons
Katie used.

Georgia Milestones Grade 4 EOG Study/Resource Guide for Students and Parents Page 137 of 152
Copyright © 2020 by Georgia Department of Education. All rights reserved.
Mathematics

Standard/ DOK Correct


Item Explanation
Element Level Answer
25 MGSE4.MD.1 2 N/A See scoring rubric and exemplar response on page 146.
26 MGSE4.G.1 2 N/A See scoring rubric and exemplar response on page 147.
27 MGSE4.MD.4 1 N/A See scoring rubric and exemplar response on page 148.

Page 138 of 152 Georgia Milestones Grade 4 EOG Study/Resource Guide for Students and Parents
Copyright © 2020 by Georgia Department of Education. All rights reserved.
Mathematics

MATHEMATICS EXAMPLE SCORING RUBRICS AND EXEMPLAR RESPONSES


Item 3
Scoring Rubric

Points Description
2 The student correctly represents the number in expanded form.
1 The student correctly represents two or three of the four digits in expanded form.
0 The student does not correctly represent at least two of the digits in expanded form.

Exemplar Response
The correct response is shown below.

The given number is read one thousand five hundred seven. There is a 1 in the thousands place, which is
equal to “1” × “1000.” There is a 5 in the hundreds place, which is equal to “5” × “100.” There is a 0 in
the tens place, which is equal to “0” × “10.” There is a 7 in the ones place, which is equal to “7” × “1.”
To represent 1,507 in expanded form, the four multiplication expressions are added together.

Georgia Milestones Grade 4 EOG Study/Resource Guide for Students and Parents Page 139 of 152
Copyright © 2020 by Georgia Department of Education. All rights reserved.
Mathematics

Item 4
Scoring Rubric

Points Description
2 The student correctly selects both drop-down menu options.
1 The student correctly selects one of the drop-down menu options.
0 The student does not correctly select any of the drop-down menu options.

Exemplar Response
The correct response is shown below.

This is the correct response for the first drop-down menu because “one thousand six hundred thirty-four”
represents the word form of 1,634. This is the correct response for the second drop-down menu because
the digits in the thousands place and the hundreds place are the same in both numbers and the digit in
the tens place in the drop-down option (4) is greater than the digit in the tens place (3) in the original
number, which means the number represented by the correct response is larger than 1,634.

Page 140 of 152 Georgia Milestones Grade 4 EOG Study/Resource Guide for Students and Parents
Copyright © 2020 by Georgia Department of Education. All rights reserved.
Mathematics

Item 8
Scoring Rubric

Points Description
1 The student correctly places all the factors.
0 The student does not correctly place all the factors.

Exemplar Response
The correct response is shown below.

This is the correct response because 12 and 16 share the factors 1, 2, and 4, while 3, 6, and 12 are
factors of 12 but not 16 and 8 and 16 are factors of 16 but not 12.

Georgia Milestones Grade 4 EOG Study/Resource Guide for Students and Parents Page 141 of 152
Copyright © 2020 by Georgia Department of Education. All rights reserved.
Mathematics

Item 11
Scoring Rubric

Points Description
1 The student correctly plots the point.
0 The student does not correctly plot the point.

Exemplar Response
Two correct responses are shown below.

1 3
There are 8 marks from 0 to 1, which makes each of the marks equal to __ ​​   ​​.   The fraction __
​​   ​​  is equivalent
6 8 4
to the fraction __
​​   ​​  , so the point needs to be placed between the third and sixth marks on the number line.
8
Plotting a point on the fourth or fifth marks is a correct response.

Page 142 of 152 Georgia Milestones Grade 4 EOG Study/Resource Guide for Students and Parents
Copyright © 2020 by Georgia Department of Education. All rights reserved.
Mathematics

Item 15
Scoring Rubric

Points Description
2 The student correctly answers both Part A and Part B.
1 The student correctly answers either Part A OR Part B.
0 The student does not correctly answer either part.

Exemplar Response
Part A
The correct response is shown below.

5 3
Each group of 5 squares represents __ ​​   ​​  , or 1 whole. The expression 4 × __
​​   ​​  can be thought of as having
3 5 5
4 groups of __
​​   ​​  parts. There are 4 groups, so shading 3 squares in each of the groups represents the
5 3
expression 4 × __ ​​   ​​.  
5
Part B
15
The correct answer is choice (B) ___ ​​   ​​.   By using a similar model there would be 5 whole strips each divided
4
into 4 parts. Shading 3 of the 4 parts in each strip would yield 3 × 5 = 15 shaded squares. Each square
1 15
is __
​​   ​​  of a whole, so there would be ___
​​   ​​  shaded squares. Choice (A) is incorrect because it represents
4 4
attempting to use the standard algorithm but multiplying both the numerator and denominator by 5.
Choice (C) is incorrect because it represents counting the number of shaded squares in the diagram and
1
not recognizing that each square represents __
​​   ​​.   Choice (D) is incorrect because it represents counting all
4
the squares in the model.

Georgia Milestones Grade 4 EOG Study/Resource Guide for Students and Parents Page 143 of 152
Copyright © 2020 by Georgia Department of Education. All rights reserved.
Mathematics

Item 20
Scoring Rubric

Points Description
2 The student correctly places two lines of symmetry.
1 The student correctly places one line of symmetry.
0 The student does not correctly place a line of symmetry.

Exemplar Response
The correct response is shown below.

A line of symmetry divides a figure into two equal pieces that would fold onto each other. The shape drawn
is a rhombus, which can be divided into two equal pieces with a segment through each pair of opposite
angles.

Page 144 of 152 Georgia Milestones Grade 4 EOG Study/Resource Guide for Students and Parents
Copyright © 2020 by Georgia Department of Education. All rights reserved.
Mathematics

Item 21
Scoring Rubric

Points Description
1 The student correctly draws a shape with 1 line of symmetry.
0 The student does not correctly draw a shape with 1 line of symmetry.

Exemplar Response
Two possible correct responses are shown below.

These responses are correct because the shape in each response has 5 sides and 1 line of symmetry. The
2 sides that are added intersect halfway between the vertical sides that are provided.

Georgia Milestones Grade 4 EOG Study/Resource Guide for Students and Parents Page 145 of 152
Copyright © 2020 by Georgia Department of Education. All rights reserved.
Mathematics

Item 25
Scoring Rubric

Points Description
2 The student correctly creates the bar graph.
1 The student correctly creates two of the three bars in the bar graph.
0 The student does not correctly create at least two bars in the bar graph.

Exemplar Response
The correct response is shown below.

Each line on the vertical axis represents 20 ounces of food. Since there are 16 ounces in 1 pound, Leo
has 16 × 20 = 320 ounces of dog food, 16 × 15 = 240 ounces of cat food, and 16 × 5 = 80 ounces of
rabbit food. All three of these values can be found labeled on the vertical axis, so the first bar stops at
320, the second bar stops at 240, and the third bar stops at 80.

Page 146 of 152 Georgia Milestones Grade 4 EOG Study/Resource Guide for Students and Parents
Copyright © 2020 by Georgia Department of Education. All rights reserved.
Mathematics

Item 26
Scoring Rubric

Points Description
2 The student correctly selects all three drop-down menu options.
1 The student correctly selects the first drop-down menu option OR correctly selects both the
second and third drop-down menu options.
0 The student does not correctly select the first drop-down menu option OR correctly select
both the second and third drop-down menu options.

Exemplar Response
The correct response is shown below.

“Obtuse” is correct for the first menu because all angles greater than 90 degrees are classified as obtuse
angles. “Greater than” and “90 degrees” are correct because they explain the reference angle that is used
for classifying angles.

Georgia Milestones Grade 4 EOG Study/Resource Guide for Students and Parents Page 147 of 152
Copyright © 2020 by Georgia Department of Education. All rights reserved.
Mathematics

Item 27
Scoring Rubric

Points Description
1 The student correctly plots all seven Xs.
0 The student does not correctly plot all seven Xs.

Exemplar Response
The correct response is shown below.

This is the correct response because it shows an X to represent each number in the table.

Page 148 of 152 Georgia Milestones Grade 4 EOG Study/Resource Guide for Students and Parents
Copyright © 2020 by Georgia Department of Education. All rights reserved.
Mathematics

ACTIVITY
The following activities develop skills in Unit 2: Multiplication and Division of Whole Numbers.
Standards: MGSE4.OA.1, MGSE4.OA.2, MGSE4.OA.3, MGSE4.OA.4, MGSE4.OA.5, MGSE4.NBT.5,
MGSE4.NBT.6

Complete the following activities with a partner.


Activity 1: Use place-value blocks to model three-digit whole numbers. Your partner should decompose the
number in at least three different ways.

Example:
317 = 3 hundreds + 1 ten + 7 + 10 + 7
= 3 hundreds + 17 + 17
= 2 hundreds + 11 tens + 7 + 110 + 7
Switch roles and repeat so that each partner models at least five numbers.
Activity 2: Make a place-value chart that extends to millions. Write a whole number with 5 to 7 digits in
the chart. Your partner should read the number aloud and write it in word form. Switch roles and repeat so
that each partner writes at least five numbers.
Activity 3: Make a multiplication chart for whole numbers 0 to 10. Say a multiplication or division fact
problem. Your partner should show how to use the chart to find the product or quotient. Switch roles and
repeat so that each partner solves at least five multiplication or division problems. Then work together to
find and describe at least five patterns in the chart.

Georgia Milestones Grade 4 EOG Study/Resource Guide for Students and Parents Page 149 of 152
Copyright © 2020 by Georgia Department of Education. All rights reserved.
Mathematics

ACTIVITY
The following activities develop skills in Unit 7: Measurement.
Standards: MGSE4.MD.1, MGSE4.MD.2, MGSE4.MD.3, MGSE4.MD.4, MGSE4.MD.5, MGSE4.MD.6,
MGSE4.MD.7

Complete the following activities with a partner.


Activity 1: Use tools such as balances, scales, meter sticks, yardsticks, rulers, analog and digital clocks,
and containers marked with cups, ounces, and liters to practice measuring objects or liquids in different
units.
Activity 2: Make two conversion charts—one with customary units and one with metric units. Each chart
should give rules for converting between at least 10 pairs of units in each system. Then choose one rule
from each chart. Use each rule to record measurement equivalents in a two-column table. Then list each
pair of equivalent measures as a number pair. For example, if you choose the rule for converting feet to
inches, your number pairs might be (1, 12), (2, 24), (3, 36), etc.
Activity 3: Write at least five word problems that involve distances, intervals of time, liquid volumes,
masses of objects, and money that can be solved using the four operations. At least two of the problems
should involve simple fractions or decimals. Trade problems with another person and solve the problems
you receive. Use diagrams in your solutions, when possible.
Activity 4: Search newspapers, magazines, or the Internet for articles or websites that mention
measurements. For each example, identify what is measured and what unit is used. Explain why you
think that unit was chosen. Then create a chart called “Measurements in Real Life” that shows real-world
benchmarks for different types of measurements and units.

Page 150 of 152 Georgia Milestones Grade 4 EOG Study/Resource Guide for Students and Parents
Copyright © 2020 by Georgia Department of Education. All rights reserved.
The following skills, marked with an asterisk (*) in Language standards 1–3, are particularly likely to require continued
attention in higher grades as they are applied to increasingly sophisticated writing and speaking .
Standard Grade(s)
3 4 5 6 7 8 9–10 11–12
L .3 .1f . Ensure subject-verb and pronoun-antecedent agreement .
L .3 .3a . Choose words and phrases for effect .
L .4 .1f . Produce complete sentences, recognizing and correcting inappropriate fragments
and run-ons .
L .4 .1g . Correctly use frequently confused words (e .g ., to/too/two; there/their) .
L .4 .3a . Choose words and phrases to convey ideas precisely .*
L .4 .3b . Choose punctuation for effect .
L .5 .1d . Recognize and correct inappropriate shifts in verb tense .
L .5 .2a . Use punctuation to separate items in a series .†
L .6 .1c . Recognize and correct inappropriate shifts in pronoun number and person .
L .6 .1d . Recognize and correct vague pronouns (i .e ., ones with unclear or ambiguous
antecedents) .
L .6 .1e . Recognize variations from standard English in their own and others’ writing and
speaking, and identify and use strategies to improve expression in conventional
language .
L .6 .2a . Use punctuation (commas, parentheses, dashes) to set off nonrestrictive/parenthetical
elements .
L .6 .3a . Vary sentence patterns for meaning, reader/listener interest, and style .‡
L .6 .3b . Maintain consistency in style and tone .
L .7 .1c . Places phrases and clauses within a sentence, recognizing and correcting

Copyright © 2020 by Georgia Department of Education. All rights reserved.


Georgia Milestones Grade 4 EOG Study/Resource Guide for Students and Parents
misplaced and dangling modifiers .
L .7 .3a . Choose language that expresses ideas precisely and concisely, recognizing and
eliminating wordiness and redundancy .
L .8 .1d . Recognize and correct inappropriate shifts in verb voice and mood .
L .9-10 .1a . Use parallel structure .
* Subsumed by L .7 .3a

Subsumed by L .9-10 .1a

Subsumed by L .11-12 .3a
APPENDIX: LANGUAGE PROGRESSIVE SKILLS, BY GRADE

Page 151 of 152


Appendix: Language Progressive Skills, by Grade
Study/Resource Guide
for Students and Parents
Grade 4
End-of-Grade

You might also like